UWorld - Surgery (New & Complete) 01/2020

Réussis tes devoirs et examens dès maintenant avec Quizwiz!

XR of the spine -Pt has adolescent idiopathic scoliosis -A) MGMT ----a) Dx tests -------1) the first step in evaluation is an XR of the spine ----b) TMT ---Based on degree of curvature (Cobb angle) -------1) Mild → Cobb angle (10-30 degrees) -----------Monitor every 6-mnths (Answer A) -------2) Moderate → Cobb angle ≥30 degrees -----------Thoracolumbosacral space (Answer C) -------3) Severe → Cobb angle ≥40-50 -----------Surgery

A 13F, presents for pre-participation sports physical. She is planning to play lacrosse, which she played the previous school year. The coach notified the mother that the girl's posture seemed abnormal. The pt has no sig PMHx & takes no meds. She has not undergone menarche. BMI is at the 50th percentile. PE shows a right-sided lumbar prominence during the forward bend test, & there is no tenderness of the spine. The remainder of the exam reveals no abnormalities w/normal neurologic function. Which of the following is the best next step in MGMT? (Reassurance & follow-up in 6 months OR XR of the spine OR Thoracolumbosacral spinal space

Infertility -Pt presents w/a varicocele → characteristics include: 1) Soft, irregular mass (bag of worms) 2) Increased size when standing; Reduction on supine positioning 3) Usually in men aged 15-25 Complications 1) Infertility & testicular atrophy → treated w/ surgery (for men that are not worried about infertility, no TMT is necessary) Intestinal strangulation -This is complication of inguinal hernia not of a varicocele

A 16B, presents d/t occasional discomfort & heaviness of his scrotum over the past several months. He has had no trauma, dysuria, or urethral discharge. The pt is not sexually active. He is afebrile, & vitals are normal. On PE, an irregular, ropy mass is noted on both sides of the upper scrotum. The mass increases in size w/Valsalva & reduces on supine positioning. There is no erythema, ulcers, or tenderness of the scrotal skin & no inguinal lymphadenopathy. The remainder of the exam is normal. Surgical correction is discussed w/the pt & his parents. The primary reason to perform surgery is to prevent which of the following complications? (Infertility OR intestinal strangulation)

Fixed teardrop pupil -A) An open globe laceration (OGL) may present with: ---1) Globe deformity ---2) Extrusion of vitreous/iris ---3) Visible entry wound -B) Additionally, OGL may be inferred when any of these deformities are visualized ---1) Peaked/Fixed Teardrop pupil ---2) Increased/decreased anterior chamber depth ---3) Reduced intraocular pressure Corneal cloudiness -Acute corneal opacification suggests angle closure glaucoma Hypopyon & Perilimbal injection -Hypopyon is a layering of inflammatory cells in the dependent portion of the anterior chamber -Perilimbal injection (AKA ciliary flush) is characterized by dilation of vasculature at the junction of the sclera & cornea -Both of these conditions are seen in a variety of infectious conditions (endophthalmitis, keratitis) & inflammatory conditions (anterior uveitis/iritis)

A 16B, presents to the ED d/t an eye injury. The pt was helping his father to mow the lawn when a flying object struck his left eye. He has had foreign body sensation, pain, excessive tearing, & decreased vision in the eye. His father noticed the eye being red but did not visualize any foreign objects. The pt has no other sig PMHx & takes no meds. In the ED, the pt appears in obvious discomfort & keeps the left eye closed. Which of the following is most suggestive of globe perforation? (Fixed teardrop pupil OR Corneal cloudiness OR Hypopyon OR Perilimbal injection)

Immobilization -Pt has symptomatic hypercalcemia (nausea, polyuria) & a decreased PTH → the most likely cause is immobilization -Pathophysio --Hypercalcemia in immobilization is likely d/t increased osteoclastic bone resorption → there is increased risk of this condition in pts w/ a high rate of bone turnover, including: --1) Young pts --2) Pts w/ Pagets Hypoalbuminemia -Pts will not have calcium-related symptoms directly related to changes in albumin (b/c only 40% of circulating Ca is bound to albumin)

A 17B, is admitted to the ED following a MVC. He is found to have extensive cerebral hemorrhage leading to a deep coma, as well as fractures of the C4 vertebra, pelvis, & right femur. Following admission to the hospital, he is intubated & central lines are placed. The pt develops acute renal failure d/t rhabdomyolysis, which improves rapidly w/ TMT. He is extubated after regaining consciousness. He also undergoes percutaneous endoscopic gastrostomy tube placement d/t persistent problems w/ swallowing. The pt is unable to ambulate d/t quadriparesis resulting from cervical cord injury. Four weeks after his initial injury, he develops nausea & polyuria. Labs show: -Ca+------------------------12.1 -Albumin-------------------3.0 -Cr, serum------------------1.4 -Phosphorus---------------3.1 -Glucose-------------------108 -PTH-----------------------9 (N 10-65) -PTrH-----------------------N/A -1,25-Dihydroxy D---------19 (N 20-60) Which of the following is the most likely cause of this pts hypercalcemia? (Hypoalbuminemia OR Immobilization)

Fibromuscular dysplasia -Characteristics include: 1) 90% cases occur in young females 2) May present w/abd bruit or sub auricular systolic bruit ---Sub-auricular systolic bruit is highly suggestive of FMD → symptoms include -----Recurrent headaches -----Pulsatile tinnitus -----TIA -----Stroke Renal artery atherosclerosis -Usually seen in older men; Unlikely in young female in absence of hypercholesterolemia Primary hyperaldosteronism -Pt would have HTN → as well as, hypokalemia & metabolic alkalosis -Bruits are not typical

A 19F, presents d/t recurrent headaches. The pt has had headaches for years but thinks they have recently gotten worse; they last several hours & remit spontaneously or after taking OTC acetaminophen. She has no nausea, vomiting, abd pain, sweating, or fever. 6-mnths ago, the pt checked her BP at a local pharmacy & was told that it was too high. She has no other sig PMHx & takes no meds. FMHx is sig for HTN & DM. BP is 175/100 on the right arm & 170/102 on the left arm, pulse is 80/min, RR 14/min. On exam, she appears comfortable & cooperative. Peripheral pulses are full & symmetric. Chest exam is unremarkable. A systolic bruit is heard under the right ear. Abd exam shows no tenderness or masses. Which of the following is the most likely cause of this pt's HTN? (Fibromuscular dysplasia OR Primary hyperaldosteronism OR Renal artery atherosclerosis)

Needle thoracostomy -When Tension pneumothorax is suspected, the order of MGMT is: 1) Decompression (needle thoracostomy) ---Performed immediately to prevent CV collapse 2) Endotracheal Intubation ED Thoracotomy -Used to resuscitate penetrating trauma pts w/ witnessed/imminent cardiac arrest -Would be used in this pt if he had gone into CV collapse

A 20M, was stabbed in the chest during a street flight. BP is 90/58, pulse is 124/min, & RR are 30/min. The pt is in severe respiratory distress. Breath sounds are present on the left & absent on the right. Heart sounds are normal. The neck veins are distended. The pt becomes obtunded during exam. Which of the following is the best next step in MGMT? (Endotracheal intubation OR ED thoracotomy OR Needle thoracostomy)

Inflammation of the plantar aponeurosis -This describes plantar fasciitis → which presents w/: --1) Heel pain that is worse w/ weight bearing --2) Pain at plantar aspect of heel & hindfoot --3) RF -----Most common in obese, middle-aged adults -----Can be in young, well-conditioned athletes that perform high-impact exercise or spend extended periods bare feet -B) Physical exam findings ---1) Pain w/ direct pressure at bottom of foot Inflammation of the achilles tendon -Achilles tendinopathy is the most common cause of exercise-induced heel pain -However, Key difference is in pain characteristics: ---1) Physical exam findings -------a) Pt would have pain w/mediolateral squeezing of the heel ----------[pt has NO pain w/this; only has pain with DIRECT PRESSURE TO BOTTOM OF FOOT] -------b) the pain & tenderness are most prominent at the posterior heel, often w/ palpable thickening of the tendon -------c) The pain is typically reproduced by passive dorsiflexion of the ankle →(rather than palpation of the heel)

A 22F, presents for eval of new-onset foot pain. The pain is located at the heel & is most pronounced when she gets up from sitting after a long period of time. The pt is a collegiate soccer player who plays both indoor & outdoor soccer. In addition, she enjoys trail running & occasionally runs barefoot along the beach. PE of the foot shows a moderately high arch w/no discoloration or deformity. Direct pressure to the bottom of the heel elicits sharp pain, but mediolateral squeezing of the heel causes no discomfort. Which of the following is the most likely etiology of the pts foot pain? (Inflammation of the Achilles tendon OR Inflammation of the plantar aponeurosis)

Posterior shoulder dislocation -Presents w/ arm of affected shoulder in an adducted & internally rotated position (w/impaired external rotation & visible flattening of the anterior aspect of the shoulder & prominence of the coracoid process -RF's ---Violent seizures (this pt) ---Electrocution Anterior shoulder dislocation -Most common form of dislocation → However, usually caused by direct blow of fall on outstretched arm -Affected arm is held slightly abducted & externally rotated

A 22F, primigravida, at 34-wks present in the ED by ambulance for sudden loss of consciousness followed by generalized shaking. The paramedics start an IV bolus of Mg2+ sulfate, which was running wide open when the pt arrived at the hospital. She says she is very tired & has blurry vision & a headache. The pt also has muscle pain & sore joints & is not able to move her right arm. BP is 145/98, pulse 112/min, RR 16/min. The pt holds the right arm adduced & internally rotated. Exam shows loss of external rotation of the right arm but no sensory deficits. Deep tendon reflexes are 2+ bilaterally,. & handgrip & wrist strength is preserved on both sides. Urinalysis shows 3+ protein. Which of the following is the most likely cause of this pts arm weakness? (Anterior shoulder dislocation OR Posterior shoulder dislocation)

Enteral nutrition via feeding tube -Critically ill pts require early nutritional support → When initiated w/in ≤48 hours EN has multiple benefits including: --Reduction of infections (pneumonia) --Reduction in mortality --Maintenance of gut integrity TPN via peripheral venous catheter -Typically given via central line -Only used in pts w/ CI's to enteral nutrition (intestinal discontinuity, prolonged ileus) b/c early initiation may increase the risk of infection (Central line sepsis)

A 22M, in the ICU is being evaluated for nutritional needs. Two days ago, the pt was brought to the hospital after a severe motorcycle collision in which he sustained splenic rupture w/associated vascular injury that required multiple abd operations. The bowel was intact & peristalsing but too edematous to allow fascial closure; therefore, the abd was left open w/ a plan for delayed closure. Temp 98.8F, BP 128/84, pulse 90/min. The pt is intubated & receiving mechanical ventilation. The abd is distended w/ an open midline incision covered by a negative pressure dressing. Associated drainage is thin & serosanguinous. Which of the following is the best next step in MGMT? (Enteral nutrition via feeding tube OR TPN via peripheral venous catheter)

Incise & drain the nasal septum -Pt most likely has a a septal hematoma → this can progress to a septal abscess and cause avascular necrosis of the septal cartilage -A) TMT ---1) Incision & drainage ---2) Anterior nasal packing ---3) Ice packs & NSAID's ---4) Antibiotics ---5) Require otopharyngeal evaluation after a few days for packing removal & assessment Insert anterior nasal packing -This should be performed AFTER incision & drainage

A 22M, presents to the urgent care clinic after being hit in the face w/a basketball. He had bleeding from both sides of his nose for about 10-mins that stopped w/ pressure. He is currently unable to breathe through his nose. PMHx is sig for allergic rhinitis treated w/oral antihistamines. Temp 98.6F, BP 110/70, pulse 70/min & RR 14/min. O2 sat is 99% on room air. On exam, there is bruising across the nose & under the eyes. Anterior rhinoscopy reveals no bleeding. There is soft, fluctuant swelling of the septum bilaterally. There is no blood in the oropharynx. Which of the following is the best next step in MGMT of this pt? (Incise & drain the nasal septum OR Insert anterior nasal packing)

Emergent thoracotomy -Pt most likely has a massive hemothorax (>1500 mL) → TMT approach is: 1) Tube thoracostomy 2) Emergent thoracotomy (in some pts) ---Initial bloody output >1,500 mL ---Persistent hemorrhage (>200 mL/Hr for > 2hrs) or continuous need for blood transfusion to maintain hemodynamic stability Contrast enhanced CT -Can assist in identifying a source of bleeding → However, only appropriate in hemodynamically stable pts

A 25M, presents to the ED d/t a stab wound to the chest. The pt was returning to his parked car when he was mugged. He tired to resist, but the assailant stabbed him once in the left chest w/a switchblade. During transport, the pt became unresponsive & required intubation. He also received 2L of normal saline. Upon arrival, BP is 80/42, & pulse 132/min. Pulse Ox is 98% on 60% O2. Wt is 176.4-lbs & Ht is 5"11". On PE, there is a fist-sized blood stain on the pt's shirt & a 3-cm linear wound in the left 5th intercostal space at the midaxillary line, w/dried blood & no active external hemorrhage. Breath sounds are diminished in the left lung & normal in the right lung. Heart sounds are normal. the jugular veins are flat. The abd is non-distended & soft to palpation. FAST reveals left-sided intrathoracic free fluid. A left-sided chest tube is placed w/immediate output of 2,000 mL of blood. Which of the following is the best next step in MGMT? (Contrast-enhanced CT OR Emergent thoracotomy)

Fibrotic intestinal stricture -Pt most likely has SBO → characteristics include: RF's --1) Hx of Crohns → most likely led to a fibrotic stricture that caused the obstruction that results from poorly controlled, severe inflammation --2) Smoking & young age (<30) are sig RF's for uncontrolled inflammation Symptoms of SBO 1) Bilious vomiting 2) Severe abd pain + inability to pass stool/flatus 3) High-Pitched (tympanic) bowel sounds TMT 1) Surgery Adynamic ileus -Presents similarly to SBO → however adynamic ileum only develops after exposure to an insult that "stuns" the bowel → such as recent intra-abd surgery or high-dose opioids Adverse RXN to infliximab -GI distress is not an AE

A 25M, presents to the ED d/t abd pain, nausea, & vomiting. The symptoms began 2-mnths ago, at which time the pain was intermittent & located in the RLQ. Since then, it has progressed & is now diffuse, severe & unremitting. The pt started vomiting bilious fluid 2-days ago but has not had hematemesis, melena, hematochezia, diarrhea, or unexpected wt loss. In addition, he has not moved his bowels in 2-days, although he frequently passes flatus. PMHx is sig for Crohn's w/ ileal involvement, which if currently managed w/infliximab & has never required surgery. The pt has used acetaminophen & ibuprofen to relive his pain & has not used narcotics. He smokes 3-4 ciggs day but does not drink or use illicit drugs. Temps 99.5F, BP 134/78, pulse 95/min, RR 18/min. On PE, the pt is awake & cooperative. Chest exam is unremarkable. The abd is distended, tympanic, & tender to palpation; there is no rebound tenderness or guarding. Which of the following most likely explains this pts presentation? (Adynamic ileus OR Fibrotic intestinal stricture OR Adverse rxn to infliximab)

Fluorescein staining -Pt suffered a high-velocity eye injury → characteristic s include: --1) Persistent pain --2) Tearing --3) Foreign body sensation -This injury carries an increased risk for corneal abrasion & open globe laceration & requires further assessment -B) MGMT ----a) Diagnostic tests -------1) Fluorescein drops → In case of corneal abrasion, fluorescein preferentially stains the corneal defect & appears yellow ----b) TMT --------1) Saline irrigation -----------a) Only if a foreign body was present, however Fluorescein drops test should be preformed FIRST Continuous saline irrigation -Used to clear a superficial foreign body, but should not be performed unless on OGL has been ruled out and/or a foreign body identified -----[Pt has NO FOREIGN body on PE]

A 26M, previously healthy, presents to the ED d/t to an eye injury. The pt was hiking earlier today when the person in front of him pushed & let go of a small tree branch, which snapped back & hit him in the face. He had sudden pain in the right eye followed by persistent foreign-body sensation & increased tearing. On PE, the pt appears in moderate distress. Right eye exam reveals no foreign bodies, but there is mild conjunctival erythema, especially around the limbus. The cornea is clear w/no blood in the anterior chamber. The pupil is small, round, & reactive to light. Visual acuity is normal. Left eye exam reveals no abnormalities. Which of the following is the best next step in MGMT of this pt? (Fluorescein staining OR Continuous saline irrigation)

Fat embolism -A) RF fat embolism include: ---1) Occurs 24-72 hrs following fracture/surgical manipulation of bones that contain abundant bone marrow (femur, pelvis) 2) Signs/Symptoms → CLASSIC TRIAD --1) Respiratory distress (tachypnea, hypoxemia) --2) Neuro dysfunction (confusion) --3) Petechial rash (However, present in < 50% of cases) --4) Thrombocytopenia --5) Pulmonary edema (mimicking ARDS) after 24-48 hrs → leading to ABG abnormalities ----↑ pH (alkalosis) ----↓ PaO2 ----↓ PaCO2 -C) Diagnostic ---1) CT angio ------ground glass opacities w/no pulmonary arterial filling defects Pulmonary contusion -Can also present W/ pulmonary edema after Trauma & w/ abnormalities on ABG - However, key differences are: ---1) Usually results from CHEST TRAUMA (FLAIL CHEST) ---2) Presents w/ respiratory failure -------(usually shown by a NORMAL PCO2 level in pts w/ hypoxemia) ---3) Presents w/ LOCALIZED pulmonary edema at the site of injury --------[pt has DIFFUSE PULMONARY EDEMA] ---4) CT angio findings --------DOES NOT present w/ diffuse, scattered pattern ground glass opacities

A 26M, w/ a Hx of seizure disorder & medication non-adherence is hospitalized after a generalized tonic-clonic seizure. A witness reports that the pt lost consciousness & fell to his right side; the seizure lasted about 2-mins. Eval in the ED reveals a low serum level of his prescribed anti-epileptic drug, levetiracetam, & a displaced right femur fracture. No other injuries or lab abnormalities are present. Levetiracetam is re-started & the pt undergoes intramedullary nailing of the fracture w/ no intra-operative complications. The next day the pt is noted to be confused. Temp 99F, BP 142/86, pulse is 102/min, RR 28/min. O2 sat is 90% while breathing room air. On PE, the pt is disoriented but has no focal weakness or sensory loss. There are no skin rashes or hematoma at the surgical site. The remainder of the exam shows no abnormalities. Labs show: -Hb---------------------12.4 -Plts---------------------103K -Leukos-----------------9K -Na----------------------136 -HCO3------------------24 -Creatinine--------------0.8 -Glucose----------------118 -pH----------------------7.47 -PaO2--------------------58 -PaCO2------------------34 CT pulmonary angio shows bilateral scattered ground-glass opacities but no filling defects w/in the pulmonary vasculature. Which of the following is the most likely cause of this pts current condition? (Fat embolism OR Pulmonary contusion)

Non-seminomatous germ cells tumor -Non-seminomatous germ cell tumors characteristics include: 1) Large mediastinal mass 2) ↑ β-hCG & ↑ AFP (diagnostic) ---Non-seminomatous germ cells tumors also produce AFP [whereas choriocarcinomas only produce β-hCG] Choriocarcinoma -Produces β-hCG; however not seen in males

A 27M, presents w/complaints of cough, chest discomfort, & dyspnea on exertion. He has a 10-lbs wt loss over the past 2-mnths. The pt has a 10-py smoking Hx. He drinks an average of 2 beers each weekend. He does not use illegal drugs & has not had multiple sexual partners. PE is unremarkable. CXR reveals a large anterior mediastinal mass. Subsequent blood works shows elevated level of β-hCG & AFP. What is the most likely Dx? (Choriocarcinoma OR Non-seminomatous germ cell tumor)

Hypocalcemia -Hypocalcemia signs include: --1) Carpal spasm (Trousseau sign) --2) Chvostek sign (cheek) --3) Paresthesias --4) Neurologic (Hyperreflexia) --5) Tetany --6) Larnygospasm --7) Seizures --8) Encephalopathy --9) HF -all in setting of high-volume blood transfusions Hypermagnesium -Can cause hypocalcemia → but only in infusions of Mg2+ such as TMT of preeclampsia/eclampsia -Symptoms include → Hyporeflexia, paralysis, apnea, cardiac arrest

A 28F, is hospitalized following a MVC. She was returning home when following a wine-tasting event when she veered off the road & rolled her car. Initial eval in the ED found severe hypotension, a hip fracture, multiple rib fractures, & evidence of BAT. The pt underwent emergent ex-lap, which revealed a liver laceration & extensive hemoperitoneum. Her condition was stabilized w/ surgical intervention, fluid resuscitation, & blood transfusions. In the early post-op period soon after extubation, the pt developed numbness in the fingertips & around her lips. She also developed forceful flexion of the wrist & abduction of the thumb while her nurse was measuring her BP. Temp is 99.3F, BP 120/68, pulse 80/min, RR are 12/min. Neuro exam shows hyperactive reflexes diffusely w/ 5/5 muscle strength in all extremities. Which of the following is the most likely Dx? (Hypermagnesium OR Hypocalcemia)

Respiratory supportive care only -The pt most likely developed a transfusion -related acute lung injury (TRALI) → Characteristics include: --1) Symptoms (Hypoxia, Dyspnea, Tachycardia, Hypotension, fever) present w/in 6 hours after transfusion initiation --2) CXR → bilateral pulmonary infiltrates TMT 1) Immediate transfusion cessation & Respiratory supportive care only (ventilatory support) --Most pts recover w/in 24-48 hours --50% will die Diuretic therapy -Used in TMT for transfusion-associated circulatory overload (TACO) --[Pt would have Hypertension → d/t to fluid overload]

A 30M, is admitted for observation following a motorcycle collision. He was hemodynamically stable at presentation, & the initial work-up revealed an isolated liver laceration. The pt has not chronic medical conditions & takes no meds. He consumes 3-4 alcoholic drinks daily & has been smoking 1-pack of ciggs daily for the past 10-years. Overnight, his Hb levels acutely decreased, & a packed RBC transfusion was ordered. An hour after the transfusion starts, the pt develops difficulty breathing. The transfusion is stopped. Temp is 98.4F, BP is 86/42, pulse is 110/min, RR are 26/min. Pulse Ox is 88% on room air. Lung exam reveals bilateral crackles. Skin exam is normal. CXR reveals bilateral pulmonary infiltrates. Which of the following is the best next step in MGMT of this pt's respiratory distress? (Diuretic therapy OR Respiratory supportive care only)

Surgical resection -A) MGMT for papillary thyroid cancer includes ---1) Surgical resection → (PRIMARY TMT) ---2) Radioactive iodine ablation -------This follows surgery -------Is warranted for pts w/ increased risk of tumor recurrence → (large tumors, extra-thyroid, LN mets, incomplete resection)

A 33F, presents for a follow-up to a thyroid nodule that was discovered incidentally on a routine PE 3-wks ago. The pt feels well & has experienced no wt changes, temp intolerance, or menstrual irregularities. PMHs is sig for primary hypothyroidism treated w/levothyroxine. FMHx is notable for HTN & hypothyroidism in her mother, but the pt has no relatives w/ a Hx of thyroid cancer. TSH is normal. Neck US reveals a 2.0-cm hypo-echoic nodule in the right lobe of her thyroid w/out any enlarged LN's. FNA reveals large cells w/ ground glass cytoplasm, & pale nuclei containing inclusion bodies & central grooving consistent w/ papillary thyroid cancer. Which of the following is the best next step in MGMT? (Radioactive iodine ablation OR Surgical resection)

Bronchogenic cyst -Located in the middle mediastinum (others mediastinal masses include: tracheal tumors, pericardial cysts, lymphoma, LN enlargement, aortic aneurysms of arch) Thymoma -Usually in anterior mediastinum -Pt would have PMHx sig for myasthenia & may present w/ hoarseness, Horner's, & facial & UE edema

A 34F, is evaluated for vague chest discomfort. She had a recent URI. PMHx is not sig. Temp 99F, BP 122/78, pulse 70/min, RR 16/min. PE findings are negative & the ECG reveals a normal sinus rhythm. CXR reveals an enlarged middle mediastinum. A helical CT reveals a mass in the middle mediastinum. What is the most likely Dx? (Thymoma OR Bronchogenic cyst)

Perilymphatic fistula -A) RF ---1) Rare complication of head trauma -B) Signs/symptoms ---1) Progressive sensorineural hearing loss ---2) Episodic vertigo w/nystagmus -------(can be triggered by Valsalva and/or elevation changes (elevator) -C) Associated Tests ---1) LOUD CLAP TEST (Tullio phenomenon) ------a) A loud clap causes pressure changes through the ossicles → which can lead to nystagmus Eustachian tube defect - DOES NOT PRESENT w/ Dizziness, nausea, or spinning sensations -A) Presentation includes: ---1) Ear popping/cracking, & hearing loss ---2) Sense of ear fullness/pain -B) Otoscopic findings ----1) tympanic membrane abnormalities -------[pt has NORMAL tympanic membranes] Orthostatic hypotension -This would cause syncope -DOES NOT PRESENT w/vertigo/nystagmus

A 34M, presents d/t intermittent dizziness over the past 3-mnths. The pt has had episodes of sudden spinning sensation, accompanied by nausea, that resolve spontaneously after approximately a minute. Symptoms occur when he is lifting heavy weights, riding an elevator, or after sneezing. He has had no headache or ear pain but has trouble hearing out of the right ear. The pt had a concussion after a bike collision 4-mnths ago but has not other sig PMHx. Vitals are normal. PE shows normal ears, including tympanic membranes. There is no extremity weakness or sensory loss. No nystagmus is present at rest but performing a Valsalva maneuver provokes nystagmus & the other reported symptoms. Which of the following is the most likely Dx? (Orthostatic hypotension OR Eustachian tube defect OR Perilymphatic fistula)

Recent head trauma -Pt most likely presents w/ CSF rhinorrhea → characteristics include: --1) Clear, unilateral rhinorrhea that increases at times of relatively increased ICP (Bending over, BM's) --2) Metallic/salty taste --3) Most often occurs after trauma (especially w/ fractures of the skull base [cribiform plate, temporal bone]) -B) MGMT ----a) Diagnostic tests --------1) Testing nasal discharge for CSF proteins (β2 transferrin, β-trace protein) ----b) TMT --------1) Inpatient MGMT ------------a) Bed rest, head elevation, frequent neuro evals) → b/c at increased risk for meningitis Use of decongestant spray - Can also result in clear nasal discharge - However, key differences are: ---1) Usually presents as bilateral rhinorrhea & severe rebound nasal congestion (rhinitis medicamentosa) -------[pt has UNILATERAL RHINORRHEA] ---2) Pt would have a Hx of excessive use of decongestant sprays -------[pt has no Hx of decongestant spray use] ---3) Pt would have swollen, erythematous turbinates -------[pt has NORMAL turbinates]

A 35F, presents d/t persistent rhinorrhea. The pt has clear drainage from the right side of her nose that increases when she has a BM or bends over to pick something off the floor. On exam, the external nose appears normal, the turbinates appear pink, & the septum is straight. There is a small amount of clear fluid from her right nostril. The remainder of the exam is normal. Which of the following additional Hx is most important for establishing a Dx? (Recent head trauma OR Use of decongestant sprays)

Epidermal inclusion cyst -A) Signs/symptoms ---1) Can occur anywhere → but most common on the face, neck, scalp, trunk ---2) They can increase in size & will usually self resolve but can recur ---3) May intermittently produce a white cheesy discharge ---4) Typically not painful (but can be) -B) PE Findings ---1) Described as dome shaped, firm, freely mobile cyst/nodule w/ a central punctum (small, dilated, pore-like opening) Lipoma - Also presents as a freely mobile bump/lump However, key differences are: ---1) DOES NOT regress & recur ---2) Present as a soft, rubbery, benign sub-Q mass w/ normal overlying epidermis

A 35M, presents d/t a 2-month Hx of a skin bump on his lower back. He noticed a similar bump in the same location about 6-mnths ago, which spontaneously resolved after 3-wks. The pt recalls no trauma preceding either occurrence. The bump has grown slightly larger in the past month, but there is no associated pruritus, pain, or other symptoms. PMHx is not sig, & he does not smoke, drink, or use drugs. Vitals are w/in normal limits. Skin exam shows a firm, mobile nodule (shown). There is no drainage from the lesion or tenderness to palpation. The lesion does not change shape when pinched at the edges. The remainder of the exam is normal. What is the most likely Dx? (Epidermal inclusion cyst OR Lipoma)

Mammography -Pt is over 30 years old → MGMT approach is: --1) Mammogram ± US Reassurance -Not correct b/c even if pt was <30 the MGMT approach would be: --1) US ± mammogram

A 36F, presents after discovering a lump in her right breast. She has no other symptoms. This pt regularly performs breast self-exams after menses, & her last menstrual period ended 2 days ago. She has no other sig PMHx & has not seen a physician in 10 years. The pt takes no meds 6 has no allergies. Her grandmother died at age 78, 2 years after being Dx'd w/ breast cancer. Breast exam shows a 1-cm firm, round mass in the upper outer quadrant of the right breast. No axillary LN's are palpable. Which of the following is the most appropriate next step in MGMT? (Mammography OR Reassurance)

Decreased adrenal hormone levels -Pt most likely has pituitary apoplexy → characteristics include: --1) RF → Hx of pituitary adenoma --2) Severe headaches --3) Ptosis & vision changes (Bitemporal hemianopsia & impaired adduction) --4) Severe orthostatic hypotension Pathophysio -The loss of pituitary hormones (particularly ACTH) can lead to adrenal crisis → which leads to ↓ cortisol & hypotension (Cortisol helps maintain vascular tone by potentiating the vasoconstrictive effects of catecholamines (NE) Increased global intracranial pressure -Can cause headaches & vision disturbance → However, would not cause severe hypotension [Associated w/ HTN & bradycardia] DA med AE (Cabergoline) - DA agonists - Used in the TMT of pituitary adenoma - AE includes MVMT disorders -----(DA agonists are not known to precipitate pituitary apoplexy)

A 36F, presents to the ED d/t severe headache & vision disturbance. The symptoms began abruptly an hour ago, & since then she has also felt weak, dizzy, & nauseated. The pt has had no similar symptoms in the past. PMHx is sig for a prolactinoma Dx'd a month ago, which is being treated w/ cabergoline. She does not smoke, drink, or use drugs. Temp is 99.7F, supine BP is 80/50, w/ a decrease to 60/40 on standing, & pulse is 110/min. On PE, the pt appears anxious & in moderate distress. There is mild ptosis & impaired adduction on the right. Bitemporal hemianopsia is present. Bilateral UE & LE muscle strength & sensation are normal. The neck veins are flat, heart & lung sounds are normal, & there is no abd tenderness or extremity edema. Which of the following is the most likely underlying cause of this pt's hemodynamic changes? Decreased adrenal hormone levels OR DA med AE OR Increased global intracranial pressure)

Clean intermittent catheterization -we are trying to prevent a catheter-associated UTI (CA-UTI) → the most effective method to prevent this is: 1) Minimizing the duration of catheterization -However, this pt has neurogenic bladder which requires long-term catheter use → the best way to avoid CA-UTI in this pt is: 1) Periodic insertion & removal (q 4-6 hrs) of a clean catheter 2) Indwelling cath for pts that can't do it on their own Daily antibacterial application to the urethral meatus -Use of antibacterial creams/washes have not been shown to decrease risk of CA-UTI

A 40M, is evaluated prior to being discharged from the hospital to a rehab center. The pt was brought to the ED 2-wks ago after his motorcycle collided w/ a truck. He was thrown 3-m (10-ft) away & sustained multiple rib & thoracic vertebrae fractures. The pt had no sensation or motor function below the T10 spinal level. Surgical stabilization of the fracture was performed, but there was no neurologic recovery. The spinal injury is also complicated by neurogenic bladder requiring urinary catheterization. Which of the following is most likely to reduce the risk of infection associated w/urinary catheter use in this pt? (Clean intermittent catheterization OR Daily antibacterial application to the urethral meatus)

Infective endocarditis -Pt presents w/ a splenic abscess → which is typically associated w/ IE → presentation of splenic abscess includes classic triad: --1) Fever --2) Leukocytosis --3) LUQ pain -----Left-sided pleuritic pain w/ left pleural effusion --4) Splenomegaly Hodgkins -Can present w/ wt loss, fever, malaise, & weakness -However, not typically associated w/ LUQ pain & splenic abscess

A 40M, presents to the ED w/ 2 wks of fever, malaise, weakness, & an unintentional wt loss. During the past 4 days, he developed left-sided chest & upper abd pain. He has asymptomatic MVP. His wife was treated for an URTI 3 weeks ago. The pt works for a large corporation & frequently travels to Mexico. He has a 20-py smoking Hx. He does not drink. Temp is 103F, BP is 120/70, pulse is 96/min, RR are 16/min. Lung exam shows decreased breath sounds in the left lower lung field w/ dullness to percussion. Cardiac exam shows a 2/6 systolic murmur at the apex. Labs show: -WBC's------------------------27K --Neutros---------------------60% --Bands-----------------------15% -Hb----------------------------13 -Plts----------------------------250K Imaging studies reveal a left-sided pleural effusion & splenomegaly w/ a splenic fluid collection. Which of the following is the most likely underlying Dx? (Hodgkins OR Infective endocarditis)

Pancreatitis -Pancreatic ascites is characterized by: ---Serosanguinous/straw-colored ---High amylase (Often >1000mL) ---High total protein (>2.5) ---Low serum-ascites albumin gradient (SAAG, <1.1, indicates portal HTN) Intestinal perforation -Labs would show: ---High (>250 cell/mm3) polymorphonuclear neutros Cts ---Low glucose ---High LDH ---Normal amylase ---High total protein ---Low SAAG -Would present w/ sudden, severe pain associated w/fever & leukocytosis (pt would have a Hx of ulcers)

A 41M, presents d/t abd pain & distention. Over the past 2-mnths, the pt has noted constant, gnawing epigastric pain that intensifies after meals. More recently, his abd has become progressively enlarged & uncomfortable. The pt has been admitted to the hospital multiple times d/t alcohol-related intox & seizures. He does not smoke, or use IV drugs. FMHx is unremarkable. Temp is 99.9F, BP 132/78, pulse 80/min. MSE is normal, & cardiopulmonary exam is unremarkable. The abd is grossly distended & mildly tender to palpation in the epigastric region. Shifting dullness is present. A diagnostic paracentesis is performed, which reveals serosanguinous fluid. Analysis of the fluid shows high level of both total protein & amylase. Serum albumin is 3.4. Which of the following is the most likely explanation for this pts presentation? (Intestinal perforation OR Pancreatitis)

Malignant hyperthermia -Pt developed: --1) Tachycardia --2) Dyspnea --3) Muscle rigidity --4) Myoglobinuria (Dark urine) --5) Hyperthermia --6) Hypercarbia --5) all soon after anesthesia (extubation means she required anesthesia for her procedure) ---15-mins post surgery Transfusion related lung injury - Also Presents w/ acute dyspnea, hypoxia, & lung infiltrates - However, key differences are: ---1) Crackles on lung auscultation bilaterally ------[pt has CLEAR lungs bilaterally on auscultation] ---2) DOES NOT PRESENT w/muscle rigidity

A 42F, is evaluated in the post-op ICU for sudden-onset tachycardia & dyspnea 15-mins after surgery. The pt underwent a dilation & curettage for gestational trophoblastic disease complicated by intra-operative bleeding that required 2 units of packed RBCs. The bleeding was controlled, & the remainder of the procedure & subsequent extubation were uneventful. The pt has a 10-py smoking Hx but no other med conditions or prior surgeries. She takes no daily meds. Temp is 100F, BP is 160/110, Pulse is 134/min, RR are 38/min. The pt appears diaphoretic & has shallow labored breathing. Cardiac exam shows sinus tachy w/ no rubs or murmurs. The lungs are clear to auscultation bilaterally. The abd has no marked distention, rebound, or guarding. Bilateral extremities have generalized rigidity. The Foley catheter is draining brown-appearing urine. Which of the following is the most likely Dx? (Malignant Hyperthermia OR Transfusion-related acute lung injury)

Esophageal perforation -The injury pattern of pedestrians that are stuck my cars often includes LE trauma (from bumper) accompanied by thoracic trauma (±head/upper extremity trauma from rolling onto hood) → Thoracic trauma can rapidly increase intraesophageal pressure enough to rupture the esophagus → symptoms include turbid green drainage (indicating esophageal perforation) Diaphragmatic rupture -Can also occur after blunt abd trauma → however, CXR typically reveals elevated hemi-diaphragm or migration of abd contents [pts CXR does not show this]

A 42M, present to the ED 35-mins after being struck by a car while crossing the intersection. The paramedics found him hypotensive & intubated him in the field for respiratory distress. After resuscitation w/2 liters of normal saline, his BP is 110/80 & pulse is 90/min. There is diffuse bruising across the chest, & the right lower leg is visibly deformed. CT scan of the head, cervical spine, & abd are normal. CT scan of the chest reveals bilateral, mildly displaced rib fractures w/underlying pulmonary contusions & a right-sided pneumothorax. A chest tube is placed. The pt undergoes open reduction & internal fixation of right tibial & fibular fractures & is extubated following surgery. One day later his chest tube is draining turbid, green fluid. Repeat CXR shows resolution of the pneumothorax but there is a new right-sided plural effusion. Which of the following is the most likely Dx? (Diaphragmatic rupture OR Esophageal perforation)

Close contact w/dogs -Pt most likely has a dog tapeworm from Eichinococcus granulosus → characteristics include: 1) Endemic to rural, developing countries --particularly high in areas of sheep farming 2) ≥ 1 Hydatid cysts ---Hepatic cysts often have daughter cysts (internal septations) -B) TMT ---1) Albendazole → small cysts ---2) Surgery → Large cysts Dysentery w/in previous few months -Describes entamoeba → often asymptomatic but may cause Stomach pain dysentery/colitis & liver abscess - However, key differences are: ---1) Stomach pain is dysentery/colitis -------[pt does not have either] ---2) fever, ---3) RUQ pain, ---4) a smooth, cystic, sub-scapular mass -----[pt is afebrile & has a cystic lesion w/daughter cysts → makes Eichinococcus more likely] Ingestion of undercooked freshwater fish -May transmit liver flukes or fish tapeworm ---Liver flukes cause → biliary disease (not hepatic cysts ---Fish tapeworms cause → rarely cause symptoms, symptoms may be associated w/ Vit B12 deficiency (megaloblastic anemia)

A 42M, presents d/t progressive upper abd discomfort & nausea over several weeks. He was a farmer in rural Guatemala before immigrating to the US 5-yrs ago. Temp is 98.4F, BP is 138/82, & pulse is 68/min. Cardiopulmonary exam is normal. Hepatomegaly is present. There is no lower extremity edema. Liver US shows a 10-cm, smooth, round cyst w/ daughter cysts inside. Which of the following is most likely to be associated w/this pt's current condition? (Close contact w/ dogs OR Dysentery w/in the previous few months OR Ingestion of undercooked freshwater fish)

Zinc -Zinc deficiency presents w/: 1) Hypogonadism 2) Impaired wound healing 3) Impaired taste 4) Immune dysfunction 5) Alopecia 6) Skin rash (erythematous pustules around body orificies (mouth) & on the extremities 7) RF → ---Malabsorption (Crohns, diarrhea) ---Parenteral nutrition Niacin -Deficiency causes Pellagra → Symptoms include: 1) Dermatitis (Described as hyperpigmented & symmetrical in sun-exposed areas [not pustular] 2) Diarrhea 3) Dementia [pt does not have dementia] [pt has Hx of Crohns]

A 42M, w/a Hx of Crohn's presents for a follow-up. He had a partial ileal resection d/t a stricture & also had multiple surgeries to treat an enterocutaneuous fistula. The pt has received parenteral nutrition for the past several weeks & recently re-started oral feeding. He reports non-bloody diarrhea but not fever or abd pain. The pt says that food does not taste the same as before. On exam, he has patchy alopecia & a pustular, crusting rash w/ scaling & erythema around the mouth & on the extremities. No abd tenderness is present & bowel sounds are normal. JVP is normal & there is no LE edema. Which of the following is most likely to improve this pts current condition? (Niacin OR Zinc)

Gallbladder stasis -Pt has RF → TPN ---The presence of proteins & Fatty acids in the duodenum act as a stimulus for release of CCK → Pts on TPN (or prolonged fasting) have decreased/absent CCK release which causes decreased/absent gallbladder contraction (stasis) Hypertriglyceridemia -Pt has not Hx of hyperlipidemia Increased enterohepatic recycling of bile acids -Decreased enterohepatic recycling would predispose this pt to gallstones -Would be seen in Crohns or ileal resection

A 44M, who had an extensive small bowel resection for Crohns has been on TPN for two years. He presented to the hospital w/ epigastric & RUQ pain. Vitals are normal. PE shows mild RUQ tenderness. An US shows several gallstones; an US performed two years go did not demonstrate gall stones. Which of the following is the most likely cause of his gallstones? (Gallbladder stasis OR Hypertriglyceridemia OR Increased enterohepatic recycling of bile acids)

Ventricular free wall rupture -A) Time frame ---1) 3-5 days post-MI -B) Signs/Symptoms (for Free wall rupture): ---1) Sudden onset chest pain ---2) Profound shock ---3) Pulseless electrical activity (PEA; Pulse is not palpable) ---4) Cardiac tamponade Ventricular aneurysm -A) Time frame ----1) 2 weeks-to-months -B) Signs/Symptoms ---1) Arrhythmias, ---2) Cardiogenic shock (HF), ---3) refractory angina, ---4) Thromboembolism Papillary muscle rupture -A) Time frame ---1) 3-5 days post-MI -B) Signs/symptoms (KEY CHARACTERISTICS) ---1) Hypotension ---2) Pulmonary edema --------[PT has no pulmonary edema] ---3) Murmurs (depends on side of rupture) -----a) LV → Mitral regurg --------Holosystolic murmur at apex -----b) RV → Tricuspid regurg --------Holosystolic murmur at left lower sternal border ---4) Unlikely to cause the rapid decompensation -------(seen in this pt)

A 45M, mildly overweight man admitted to the hospital 5-days ago w/ anterior wall MI develops sudden onset left-sided chest pain. He underwent cardiac catheterization on admission that showed complete occlusion of the LAD. However, no revascularization was performed d/t late presentation. He has had no dyspnea, extremity swelling, or palpitations since admission. PMHx is sig for DM II & hyperlipidemia. He appears agitated & restless. Two mins later, he is unresponsive. Pulse is not palpable & EEG shows sinus tachycardia of 130/min. Which of the following is the most likely Dx? (Papillary muscle rupture OR Ventricular aneurysm OR Ventricular free wall rupture)

Celiacs -Pts w/ celiacs are at increased risk for bone fractures b/c of vit D malabsorption Hypothyroidism -HYPER-thyroidism will increase the risk of fractures (not hypothyroid)

A 45M, presents to the ED d/t left wrist pain after a fall. The pt was walking in his house when he dropped over a rug & landed on his outstretched hand. His wrist swelled & he applied ice, which did not improve the swelling or pain. PMHx is notable for celiacs, HTN, & hypothyroidism for which he takes appropriate meds. The pt does not smoke, drink, or use drugs. He has NKA. BMI is 20. Exam shows a "dinner-fork" deformity at the left wrist. The wrist is swollen & tender to palpation. Sensation over the left hand is intact, & the pt is able to move all his fingers. XR of the wrist is shown. Which if the following is the most sig RF for bone fracture in this pt? (Celiacs OR Hypothyroidism)

Teeth roots -Pt presents w/Ludwig angina → described as a rapidly progressive cellulitis of the submandibular space -Characteristics include: --1) Most cases are polymicrobial --2) Most cases arise from dental infections --3) Pts present w/ -----Systemic symptoms (fever, malaise, chills) -----Local compressive symptoms (mouth pain, drooling, dysphagia, muffled voice, airway compromise) Epiglottis -Can present w/ muffled voice, drooling, & airway compromise -However, Pt has swelling on the floor of the mouth & in the submandibular area [these are not epiglottic/supraglottic structures]

A 54M, presents to the ED after 2 days of fever, chills, dysphagia, & drooling. The pt has been unable to eat or drink d/t pain in his mouth & neck. He has a Hx of heavy alcohol use but does not use recreational drugs. Temp is 101.8F, BP is 110/70, pulse 108/min, RR 22/min. The pt appears toxic w/drooling & a muffled voice. The tongue is displaced posteriorly & superiorly d/t a swollen area on the floor of the mouth. The bilateral submandibular area is tender, indurated, & non-fluctuant w/palpable crepitus. Which of the following is the most likely source of infection in this pt? (Epiglottis OR Teeth roots)

Hemobilia -The pt most likely has Hemobilia (bleeding into the biliary tract) -A) RFs include: ---1) Recent liver biopsy ------(most commonly presents ~5 days after biopsy) ---2) Cholecystectomy ---3) ERCP ---4) Hepatobiliary tumors ---5) Blunt trauma -B) Signs/symptoms --1) Melena, hematemesis (UGIB → Upper GI bleed) --2) RUQ pain --3) Jaundice --4) Anemia --5) Hyperbilirubinemia -C) MGMT ----1) Conservative MGMT → (Usually self-limited) Aortoenteric fistula -Can also present w/ GI bleeding -However, key differences are: ---1) Usually occurs as a result of ruptured AAA or surgery -------[not a typical complication of liver biopsy] ---2) DOES NOT PRESENT w/ hyperbilirubinemia & jaundice

A 45M, presents to the ED for colicky, RUQ pain; vomiting; & melena for 24 hrs. He has no hematemesis. PMHx is sig for non-alcoholic steatohepatitis, which was Dx'd by percutaneous liver biopsy 5-days ago. The biopsy showed mild fibrosis around portal tracts w/out bridging fibrosis or regenerative hepatocyte nodules. Temp is 99.3F, BP is 95/60, pulse is 110/min, RR are 15/min. Scleral icterus is present. Cardiopulmonary exam is normal. The abd is tender to palpation in the RUQ. Labs show: -Hb---------------------8.7 -Plts---------------------545K -Leuko's-----------------12K -Albumin----------------3.8 -Total bili----------------4.1 What is the most likely Dx? (Aortoenteric fistula OR Hemobilia)

Pancreatic enzyme inactivation -Pt most likely has ZES → Characteristics include: --1) Multiple peptic ulcers (duodenal & jejunal) that are refractory to TMT --2) Chronic diarrhea --3) Ulcers distal to duodenum Pathophysio -The excess gastric acid in the SI can cause diarrhea & steatorrhea by inactivation of pancreatic enzymes & injury to the mucosal brush border Reduced bile acid absorption -Typically seen in pts w/ a Hx of Crohns & resection if ileum --[Crohns pts do not typically have multiple ulcers] Autoimmune mucosal injury -Can describe either celiac or pernicious anemia → However : ---Celiacs presents w/ steatorrhea ---Pernicious anemia does not present w/steatorrhea -Pt does not have celiacs b/c celiacs does not present w/ multiple ulcers -Also, Crohns presents w/ other symptoms (diarrhea) & would be most likely in a younger pt

A 45M, presents w/ a 6-mnths Hx of recurrent, burning epigastric pain & diarrhea. His stools are frothy & unusually foul smelling & they float. The pts clothes are fit loosely, & he believes that he may have lost some wt. He has tried several OTC meds, including antacids, H2 blockers, & PPI's, w/ moderate success. Vital signs are normal. Exam shows no abnormalities expect for mid-epigastric tenderness to deep palpation. Test of the stool for occult blood is positive, & stool fat is also positive. GI endoscopy reveals two duodenal ulcers & a jejunal ulcer. Which of the following is the best explanation for this pt's impaired fat absorption? (Autoimmune mucosal injury OR Pancreatic enzyme inactivation OR Reduced bile acid absorption)

Empyema - The pt most likely has pneumonia → complicated by empyema -A) Criteria for "complicated" ---1) Pleuritic chest pain ---2) Large left-sided pleural effusion ------(whole left lung is affected; white on CXR) suggests that the pneumonia is complicated by a empyema. ---3) Worsening fever ---4) Decreased fremitus ---5) Decreased breath sounds ---6) Dullness to percussion -B) RF ---1) Approximately 40% of pneumonias are associated w/ pleural effusions → most are small parapneumonic effusions but some may become large & complicated (AKA empyemas) -C) TMT ---1) Prolonged antibiotics (2-4 wks) ---2) Drainage w/ chest tube (thoracostomy) Lung abscess - Can also present w/ Productive cough & SOB - However, key differences are: ---1) Usually presents in pt w/ these specific RFs ------(obtunded Alcoholic, Pt w/ seizure disorder, Hx of infective endocarditis, pt w/ incompletely treated pneumonia) --------[pt does not meet any of these criteria] ---2) More common in RIGHT lung --------[pts LEFT LUNG is affected] ---3) Fever + Productive cough w/ Foul smelling sputum that develops over days/weeks ---------[pt does not have foul smelling sputum] ---4) CXR findings -------1) would show marked by a cavity w/ air fluid levels --------[Pts CXR shows Large left-sided pleural effusion] P.jiroveci - Can also present w/ Productive cough & SOB However, key differences are: ---1) P.jiroveci usually only presents in SEVERELY ILL/IMMUNOCOMPROMISED PTS ------[pt does not meet criteria] ---2) CXR findings -----1) Bilateral interstitial/alveolar infiltrates ------[Pts CXR shows Large left-sided pleural effusion]

A 45M, w/ HIV, presents to the ED d/t increasing SOB, left-sided chest pain, chills, & productive cough. The symptoms began 7-days ago. At first, the pt thought he had a flu, but after 3-days he began coughing up green phlegm & developed left-sided chest pain. The pain is worse w/ deep breaths or cough. CD4 Ct is unknown. He smokes & drinks regularly. Temp is 102F, BP is 110/70, pulse is 110/min, RR are 26/min. PE shows poor dentition & decreased breath sounds over the left lung base. CXR is attached. Which of the following is the most likely cause of this pts current symptoms? (Empyema OR Lung abscess OR P.jiroveci)

Symptomatic TMT only & close observation -Pt presents w/ post-op fever → to determine the TMT approach, we must determine the type to post op fever: 1) Immediate ---a) Fever develops w/in hours of operation ------Most cases caused by tissue damage (open cholecystectomy) ---b) Fever & leukocytosis lasts <3 days --MGMT ----a) Symptomatic TMT (acetaminophen) 2) Acute ---a) Fever develops in 1-7 days post-op ---b) Nosocomial infection (UTI, pneumonia) --MGMT ----a) Antibiotics 3) Subacute ---a) Fever develops in 7-28 days ---b) Abscess/infection of a central line or surgical site --MGMT ----a) Remove surgical line ----b) Antibiotics Continuous positive airway pressure & chest physiotherapy -CF TMT

A 48F, presents for an elective cholecystectomy to treat symptomatic cholelithiasis. She has had increasingly severe, intermittent biliary colic pain, & US revealed several gallstones. The pt has a Hx of HTN & obesity. Pre-operative med eval & labs were unremarkable. The pt receives cefazolin for surgical site infection prophylaxis, & surgery is performed under general anesthesia. A Foley is placed preoperatively. The planned laparoscopic procedure is converted to open approach d/t difficult anatomy; otherwise, the surgery is uneventful w/minimal blood loss. Two hours later, the pt is noted to have a temp of 102F, BP is 140/85, pulse is 104/min, RR 18/min. The pt is awake & alert but reports mild pain at the surgical site. A right-sided internal jugular CV catheter, which was placed in the OR, is in good position w/no redness at the insertion site. The lungs are clear to auscultation, & heart sounds are normal. The surgical site is clean w/ minimal serosanguinous discharge. There is no LE edema. Current labs show: -Hb----------------------10.4 -Plts----------------------240K -Leukos------------------14K --Neutros----------------80% -NA+---------------------136 -K+-----------------------4.4 -HCO3------------------26 -Creatinine--------------0.8 -Total bilirubin-----------1.3 -ALP----------------------70 -AST----------------------37 -ALT----------------------49 -pH-----------------------7.4 -PaO2--------------------86 -PaCO2------------------34 CXR reveals decreased lung volumes & small areas of subsegmental thickening bilaterally. Which of the following is the most appropriate next step in MGMT? (Continuous positive airway pressure & chest physiotherapy OR Symptomatic TMT only & close observation)

Administer high dose corticosteroids -Pt most likely has acute cellular rejection (ACR) → characteristics include: 1) Recent liver transplant & now has fevers, malaise, & elevated LFT's 2) Histological findings (Classic) ---Mixed inflammatory infiltration → Portal tracts w/ eosinophils, neutrophils, & lymphocytes ---Interlobular bile duct destruction (referred to as non-suppurative changes) ---Endotheliitis → Lymphocytic sub-endothelial invasion of the portal & hepatic veins (most reliable sign of ACR) TMT 1) High dose corticosteroids 2) Thymoglobulin (steroid refractory) 3) Repeat transplant Discontinue mycophenolate -Most common AE's are --1) Cytopenias (severe neutropnenia) --2) GI upset

A 48M, presents to the ED d/t fever & malaise for the past 2-days. He also complains of fatigue & lethargy but not nausea, vomiting, diarrhea, or dysuria. PMHx is sig for alcoholic cirrhosis, which was treated w/ an orthoptic liver transplant 10-wks ago. Current meds include tacrolimus, mycophenolate mofetil, & low-dose prednisone. Temp 100.6F, BP 125/82, pulse 80/min, RR 12/min. The pt is A&O x's 3. Scleral icterus is present. Cardiopulmonary exam is unremarkable. There is a well-healed surgical scar on the abd but no tenderness or distention. Labs show: -Hb-------------------------12.9 -Leukos---------------------6K -Plts-------------------------345K -Total bili--------------------3.2 -ALP-------------------------378 -AST-------------------------345 -ALT-------------------------473 A liver biopsy shows inflammatory infiltration of the portal tracts consisting of lymphocytes, eosinophils, & neutrophils. Interlobular bile duct destruction & prominent endotheliitis are both present. Which of the following is the best next step in MGMT? (Administer high dose corticosteroids OR Discontinue mycophenolate)

Splenic abscess -A) RF (for development of abscess) ---1) Recent laparoscopic cholecystectomy ---2) HIV ---3) Heme malignancy ---4) DM -B) Signs/symptoms ---1) High persistent fever ---2) ± Tender splenomegaly ---3) LUQ pain ---4) ) Anorexia & wt loss -C) Labs ---1) Leukocytosis w/left shift -D) MGMT -----a) Diagnostic --------1) CT of abd (diagnostic) ---------2) CXR → elevated diaphragm and/or pleural effusion -----b) ) TMT ----------1) Antibiotics + splenectomy (b/c pts usually fail percutaneous aspiration) Splenic infarction -Also Presents w/ LUQ pain, fever, chills, & anorexia -However, key differences are: ---1) DOES NOT usually present w/ splenomegaly ---2) Most pts have an associated conditions that are RF for infarction including: ------a) underlying hypercoagulable disorder, ------b) a source of embolic disease (A-fib), ------c) myeloproliferative neoplasm, ------d) hemoglobinopathy (Sickle cell) ---------[pt has NONE of these conditions] Splenic vein thrombosis - Also Presents w/ LUQ pain, chills, & anorexia -However, key differences are: ---1) DOES NOT typically present w/ splenomegaly ---2) Usually occurs in the setting of portal HTN from liver cirrhosis or pancreatitis ---3) DOES NOT present w/ High fever & wt loss

A 49F, presents to the Ed d/t 2-wks of worsening left-sided abd pain, fever, & chills. The pain is constant, deep, & radiates to the back. The pt also has had decreased appetite & notes a 5-lbs wt loss during this period. She has no dysuria, urinary frequency, hematuria, diarrhea, or constipation. The pt underwent laparoscopic cholecystectomy 2-wks ago for acute calculus cholecystitis. She has a PMHx sig for DM II, HTN, & hypothyroid. Temp 103.1F, BP 130/80, pulse 102/min, RR 16/min. Cardiopulmonary exam shows no abnormalities. The spleen is enlarged & tender; there is no hepatomegaly. Bowel sounds are present. The remainder of the exam is normal. Which of the following is the most likely Dx? (Splenic abscess OR Splenic infarction OR Splenic vein thrombosis)

Renal vein thrombosis - Presents w/ -1) Flank pain & hematuria -2) Can occur in setting of acquired hypercoagulability ---Associated w/ malignancy (pancreatic cancer) -3) Enlarged kidney on US ---(Doppler → reversal of diastolic flow) Renal infarction - Most commonly in the setting of cardio-embolic disease (A-fib) - Would show a wedge-shaped imaging - HTN - Flank pain + Abd pain

A 50F, presents to the ED d/t increasing right flank pain, nausea, 7 hematuria. The flank pain started 3 -days ago and is constant, dull, & non-radiating. She reports no dysuria. The pt was Dx'd w/ pancreatic cancer a month ago & received the first cycle of chemo 2-wks ago. She has smoked a pack of ciggs daily for 30-yrs but does not drink. Temp is 98.6F, BP is 130/90, pulse is 88/min, RR 16/min. Exam shows right flank pain & CVA tenderness. Bowel sounds are present. Peripheral pulses are normal & there is no edema. Urinalysis shows: -Protein---------------------N/A -Blood----------------------Moderate -Glucose--------------------N/A -Leukocyte esterase--------N/A -Nitrites---------------------N/A -Bacteria---------------------N/A -WBC's-----------------------1-2 -RBC's------------------------20-30 -Casts------------------------N/A -Crystals----------------------N/A Abd US shows an enlarged right kidney & normal left kidney; there is no evidence of hydronephrosis. The pancreatic mass is unchanged from prior scans. Which of the following is the most likely Dx? (Renal vein thrombosis OR Renal infarction)

Vestibular schwannoma -A) Characteristics: ---1) Asymmetric/unilateral sensorineural hearing loss ---2) Imbalance ------(especially when deprived of visual input, i.e., at night) ------( NO Vertigo → not usually seen b/c the tumor develops slowly ---3) Facial numbness (as tumor enlarges) -B) Test results ---a) Tuning fork -----1) Lateralization to UN-affected ear -----2) AC > BC in both ears Vertebrobasilar insufficiency - Can also present w/ hearing loss and facial numbness & imbalance - However, key differences are: ---1) Vertebrobasilar insufficiency is most often caused by a stroke → the pt would present w/ additional numerous MSK deficiencies; an Isolated hearing loss is rare ------[pt has no MSK deficiencies] ---2) Facial numbness is peri-oral ------[Pt presents w/ hemi-facial numbness] ---3) Diplopia, dysarthria, ataxia Menieres -Also presents w/ asymmetric sensorineural hearing loss - However, key differences are: ---1) Presents in Discrete episodes of spinning vertigo, hearing loss, & aural fullness ------[pt has had PROGRESSIVE hearing loss] ---2) DOES NOT present w/ facial numbness

A 52M, presents for eval of hearing loss in the left ear. The pt works in sales & is frequently on the phone. Over the past several months, he has had difficulty hearing phone convos in the left ear & must now move the headset to the right. He has no pain or vertigo but sometimes feels off-balance when walking at night & holds onto the walls. Vitals are normal. PE reveals normal tympanic membranes w/no middle ear effusion on either side. A tuning fork placed on the forehead is heard best in the right ear. AC>BC bilaterally. There is decreased sensation to touch on the left side of the face. Bilateral UE & LE strength, deep tendon reflexes, & sensation are normal. Which of the following is the most likely cause of this pts symptoms? (Veterbrobasilar insufficiency OR Vestibular schwannoma OR Menieres)

Immediate wound exploration -Pt most likely has an expanding neck hematoma → characterized by Enlarging fluid collection (ballotable neck swelling) that develops after thyroidectomy MGMT 1) Decompression of the airway via immediate evacuation of the hematoma with wound exploration Urgent cricothyrotomy -Because this pt most likely has a hematoma, removal of the hematoma should be sufficient to improve the airway

A 53F, is evaluated in the post-anesthesia care unit following thyroid surgery. She underwent a total thyroidectomy for a retrosternal goiter that was causing dysphagia from esophageal compression. The pt states that her neck feels a little "tight" but has not sig pain or difficulty breathing. PMHx is sig otherwise unremarkable. Temp 98.6F, BP is 126/86, pulse 100/min. O2 sat is 99% on room air. On PE, the pt appears comfortable, w/no distress. Pupils are equal, round, & reactive. Voice is normal. Neck incision is well-approximated, & there is a 4-cm ballotable swelling under the incision. The lungs are clear to auscultation. Heart sounds are normal. The neck swelling increases in size during the exam. Which of the following is the best next step in MGMT? (Immediate wound exploration OR Urgent cricothyrotomy)

CT scan of abd - Pt has signs of polycythemia → there are two types: -A) Primary ------Caused by malignant transformation of RBC progenitor cells -B) Secondary ------Typically d/t elevating circulating EPO levels ------Most commonly caused by chronic hypoxia --------a) (Cardiopulmonary disease, obstructive) ---------b) EPO-producing tumors (RCC) → -C) Labs associated w/ EPO-producing tumors ----1) ↑ EPO levels ---------(The pts elevated EPO suggests secondary polycythemia d/t RCC) BM aspiration -Used to Dx primary polycythemia - However, key difference is: ---1) LAB FINDINGS ---------Primary polycythemia presents w/ ↓↓ EPO levels

A 54F, is admitted to the hospital w/ a day of right-sided weakness. The stroke team was activated, but no intervention was performed d/t long length of time since symptoms onset. The pt also notes 6-wks of headaches & generalized weakness. She has not had recent fever, SOB, cough, chest pain, palpitations, or syncope. She has not other sig PMHx & does not take meds. The pt smoked a pack of ciggs daily for 20 years but quit 15-yrs ago. She does not use alcohol or illicit drugs. Temp 98.8F, BP 160/96, pulse 80/min & regular. O2 sat is 99% on room air. BMI is 25. Cardiopulmonary exam shows no abnormalities. Abd is soft & tender; there is no hepatosplenomegaly. Extremities have no cyanosis, clubbing, or edema. A right facial droop is present. Motor strength is 2/5 in the RUE & 3/5 in the RLE & 5/5 in both the LUE & LLE. Sensation is intact diffusely. Labs show: -Hb---------------------------20.2 -Hct--------------------------61% -Leukocytes------------------7,2K -Plts--------------------------180K -Creatinine-------------------1.2 -LDL cholesterol-------------120 -EPO--------------------------22.9 (N 4.1-19.5) MRI of the brain reveals an acute infarction involving the left internal capsule. CXR is normal. ECG shows normal sinus rhythm. ECHO shows no abnormalities. Which of the following is most likely to give the Dx in this pt? (BM aspiration OR CT scan of abd)

Increased urine ammonia production -Characteristics that suggest Mg+ ammonium phosphate (Struvite) stones are: --1) Large, irregular kidney stone --2) Fever --3) Hx of recurrent UTI's Decreased urine absorption of uric acid -Associated w/ conditions of increased uric acid production (myeloproliferative disorders) -Stones are typically small, uncommon, & -Less likely to be seen in the context of recurrent UTI's

A 55F, presents d/t fever, chills, left flank pain, & dysuria for 2-days. She was treated for a UTI twice in the past 3-mnths, the latest being 4-wks ago. Both times, the pt was compliant w/ antibiotics & her symptoms improved. She has not other sig PMHx & takes no other meds. Temp 102.2F, BP 110/70, pulse 90/min, RR 16/min. BMI is 40. Exam shows left CVA tenderness. Labs show: -Hb-------------------------12 -Leukos---------------------13K Urinalysis -Blood----------------------moderate -Glucose--------------------neg -Leukocytes esterase------pos -Nitrites---------------------pos -WBC's----------------------20-30/hpf -RBC's-----------------------20-30/hpf -Casts-----------------------N/A Kidney US shows a 1.3-cm, irregularly shoaled stone in the left renal pelvis. Which of the following is the most likely mechanism of stone formation in this pt? (Decreased renal absorption of uric acid OR Increased urine ammonia production)

Bronchial mucus plug -The pt presents w/ atelectasis → Characteristics include: --1) One of the Most common complications of surgery --2) CXR shows a collapsed lung (opacification) --3) PE findings include decreased breath sounds & dullness to percussion Diaphragm perforation -More common in the setting of BAT -CXR would show small bowel in thorax [not opacification]

A 55M, is evaluated for new-onset SOB after a right hemi-colectomy 3 days ago. He has no chest pain, cough, or hemoptysis. The pt came to the hospital 4 days ago w/ a Hx of constipation, wt loss, & abd distention. He was found to have an obstructing cecal mass & underwent right hemi-colectomy w/ ileocolic anastomosis. The pt has no sig PMHx other than a 35-py smoking Hx. Temp 99F, BP 120/80, pulse 98/min, RR 22/min. Pulse Ox is 88% on room air. Lung exam reveals dullness to percussion & absence of breath sounds on the left. CXR is attached. CXR at the time of admission was normal. Which of the following is the most likely cause to this pts pulmonary findings? (Bronchial mucus plus OR Diaphragm perforation)

Nasopharyngeal carcinoma -Characteristics include: 1) RF's ---Asian descent (EBV endemic to Southern China) ---Salt-rich food 2) Nasal congestion w/ intermittent epistaxis ---± presence of soft tissue mass 3) Headache 4) Facial numbness (tingling over mid-face that progressed to complete numbness) 5) Serous otitis media 6) Early METs to bilateral cervical LN is common Nasal polyposis -Seen in pts w/ chronic sinusitis, asthma, or aspirin sensitivity → & may obstruct nasal cavity resulting in congestion & thick rhinorrhea -However, CN involvement is unlikely -Exam would reveal a gray mass w/inflammatory material

A 55M, presents d/t left-sided facial numbness. His symptoms began a month ago w/tingling over his left mid-face & have progressively worsened to complete numbness on that side. The pt has also had 2-mnths of headaches, nasal congestion w/intermittent epistaxis, & left ear fullness. He has had no fever, rhinorrhea, or purulent nasal discharge. The pt has taken OTC allergy meds, but these have provided no relief. PMHx is otherwise insignificant & he does not smoke, drink, or use drugs. He recently immigrate to the US from China to be closer to his son. Temp is 98.6F, BP 126/80, pulse 84/min. Exam reveals sensory loss to touch & pain on the left side of the face. No facial muscle weakness or other neurological findings are present. Enlarged, nontender, & mobile cervical LNs are palpable bilaterally. No oropharyngeal or nasal lesions are visualized. Otoscopy of the left ear demonstrates clear fluid behind the retracted tympanic membrane, but the right ear is normal. Nasopharyngoscopy reveals a soft-tissue mass in the nasopharynx. Which of the following is the most likely Dx? (Nasal polyposis OR Nasopharyngeal carcinoma)

Recurrence of thyroid malignancy -Thyroglobulin (Tg) is the precursor to active thyroid hormones (T4 & T3) → pt's who have undergone total thyroidectomy & radioactive iodine TMT should have no residual normal thyroid tissue and thus no Tg → therefore ↑Tg levels in this pt are likely d/t recurrent differentiated thyroid cancer AE of radioactive iodine TMT -AE's include: ---1) Sialadenitis (salivary gland infection) ---2) Dry mouth ---3) Pulmonary fibrosis -------[thyroglobulin levels are not affected as an AE of radioactive iodine TMT]

A 55M, presents for a follow-up. 6-mnths ago, the pt was Dx'd w/ papillary thyroid cancer & underwent total thyroidectomy, followed by radioactive iodine TMT. Since then, he has been taking levothyroxine & has had no issues. The pt has no other sig PMHx & does not smoke, drink, or use drugs. Vitals are normal. PE shows a well-healed surgical incision w/no palpable neck masses or enlarged cervical LN's. Lab show significantly elevated serum thyroglobulin conc. compared w/ the previous level 6-mnths ago. Which of the following best explains the observed lab findings in this pt? (AE of radioactive iodine TMT OR Recurrence of thyroid malignancy)

Acute diverticulitis -A) Signs/Symptoms: ---1) LLQ pain ---2) Low-grade fever ---3) Nausea & vomiting/anorexia ---4) Bladder symptoms ---5) Alteration in bowel habits --------(pt hasn't had a BM) -B) Associated labs ---1) Sterile pyuria ------(neg for nitrite) -C) RFs ----1) Age ----2) Obesity ----3) Smoking Acute bacterial cystitis -Also Presents w/ positive leukocyte esterase, urinary symptoms, & fever - However, key differences include: ---1) Marked Dysuria ------[pt has increased URGENCY but NO DYSURIA] ---2) Presents w/ Suprapubic pain ------[pt has LLQ pain] ---3) LABs ------1) Nitrite Positive ----------[pts labs were nitrite negative] Ovarian torsion -Also Presents w/ pelvic pain, nausea/vomiting & (occasionally) fever - However, Key differences include: ---1) Ovarian torsion presents w/ ACUTE & SEVERE pain ------[pt has DULL, PERSISTENT PAIN] ---2) Most pts present w/a palpable adnexal mass ------[Pt does not have this symptom]

A 56F, presents d/t abd pain & nausea The pt has had 2 days of dull, persistent pain in the left lower abd that has progressively worsened. She has had no appetite & vomited once today. In addition, the pt has had mild urinary urgency w/no dysuria or hematuria. She has not had a BM during this period but is passing flatus. PMHx is sig for HTN, DM II, & an ovarian cyst. Temp 100.4F, BP is 134/82, pulse is 84/min. BMI is 33. PE shows LLQ abd tenderness w/no rebound tenderness or guarding. No palpable abd mass or organomegaly is present. Bowel sounds are normoactive in all quadrants. The remainder of the exam shows no abnormalities. Urine dipstick is positive for leukocyte esterase & negative for nitrite. Which of the following is the most likely Dx? (Acute bacterial cystitis OR Acute diverticulitis OR Ovarian torsion)

Bladder rupture -Pt most likely has intraperitoneal bladder rupture → characteristics include: --1) Hx of blunt lower abd trauma --2) Inability to void --3) Abd distention w/ascites on exam (dullness to percussion, positive fluid wave) --4) Elevated BUN & creatinine Diagnostic -Retrograde cystography Rhabdomyolysis -Would cause AKI w/ elevated BUN & creatinine → however, dark urine is not typical & ascites is not expected (pt would not have abd enlargement)

A 56M, presents to the ED d/t worsening abd discomfort & distention. The discomfort is diffuse & began 1-day prior, after the pt woke up from a night of heavy drinking that included a fall in which he struck his lower abd. He has also had increasing abd girth over the past day. PMHx includes alcohol use disorder & HTN; there is a remote Hx of appendectomy. ROS is neg for fever or vomiting but positive for inability to void. Temp 98.1F, BP 158/90, pulse 90/min. Heart & lung sounds are normal. The abd is distended, mildly tender to palpation diffusively, & dull to percussion w/ positive fluid wave. Labs show: -Hb-----------------------13.2 -Plts----------------------240K -Leukos-------------------11,2K -BUN----------------------50 -Creatinine----------------3.5 -Total protein-------------6.5 -Albumin------------------4.1 -Total bili------------------0.9 Which of the following the most likely Dx? (Bladder rupture OR Rhabdomyolysis)

Foot imaging for osteomyelitis -Pt most likely has a Diabetic foot ulcers d/t long-standing diabetic neuropathy -B/c diabetic foot ulcers are colonized by a wide range of organisms, they are often complicated by adjacent soft tissue infection and/or underlying osteomyelitis Diagnostic -Criteria for Foot imaging for diabetic foot ulcers includes: ---1) Deep (exposed bone, positive probe-to-bone testing) ---2) Long-standing (present >7-14 days) ---3) Large (≥2-cm) ---4) Associated w/ increased ESR & CRP ---5) Associated w/ adjacent soft tissue infection Lower extremity angiography -This would be used in suspected PAD → However, only after ankle-brachial index testing -Additionally, we would still test for osteomyelitis first Wound dressing w/a topical antibiotic -Not until osteomyelitis is ruled out

A 58M, presents d/t an ulcer on the sole of his right foot. The pt noticed the wound 4-wks ago. He has no Hx of trauma to the area. The ulcer has failed to heal despite wound care w/moisture-retentive dressings & pressure offloading w/orthotic devices. The pt has had no right foot pain, redness, swelling, fever, or chills. He has a 10-yr Hx of DM complicated by diabetic neuropathy & nephropathy. The pt also has HTN & hyperlipidemia. The pt does no smoke, drink, or use drugs. Vitals are normal. PE shows a 3-cm ulcer under the first metatarsal head of the right foot; the wound has a clean base & no sig discharge. There is no surrounding erythema or areas of fluctuance or tenderness. Which of the following is the best next step in MGMT of this pt's ulcer? (Foot imaging for osteomyelitis OR Lower extremity angiography OR Wound dressing w/ a topical antibiotic)

Bladder cancer -Pt has many symptoms associated w/ bladder cancer, including --1) Smoking Hx & age >40 --2) Subacute voiding symptoms (dysuria, urgency, frequency) --3) Suprapubic pain (usually indicates advanced tumor) --4) Hematuria Chronic bacterial prostatitis -Urinalysis would show pyuria & bacteriuria -Prostate would be boggy & tender -Does not typically present w/ Hematuria i

A 59M, presents d/t a month of dysuria, urinary urgency, & frequency. Over the past few days, he has also had dull, non-radiating suprapubic pain. He has never had similar symptoms before. The pt has a Hx of DM II, GERD, & MDD. He has smoked a pack of ciggs daily for 40-yrs but does not drink or use drugs. Vitals are normal. The abd is soft w/no organomegaly. Mild suprapubic tenderness is present. Rectal exam reveals a smooth, firm enlargement of the prostate w/no tenderness, induration, or asymmetry. Labs show: Urinalysis -Specific gravity----------------1.016 -Protein-------------------------N/A -Blood--------------------------Moderate -Leukocyte esterase------------neg -Nitrites--------------------------neg -Bacteria------------------------N/A -WBC's--------------------------1-2/hpf -RBC's---------------------------20-30/hpf -Casts---------------------------N/A Which of the following is the most likely explanation for this pt's symptoms? (Bladder Cancer OR Chronic bacterial prostatitis)

Levofloxacin -Pt has acute bacterial prostatitis → symptoms include: 1) Tender, swollen prostate 2) Fever 3) Dysuria 4) Leukocytosis 5) Acute urinary retention (w/increased volumes seen w/ catheter) 6) Suprapubic pain TMT 1) Prolonged TMT (~6-wks) w/ fluoroquinolone (levofloxacin) OR TMP-SMX --Both can achieve high prostatic tissue levels Nitrofurantoin - Medication used for UTI - UTI can also present w/ frequency, dysuria, & suprapubic pain - However, key differences are: ---1) Prostate is NOT AFFECTED ------[pt has a tender swollen prostate] ---2) Usually presents w/ INCREASED FREQUENCY ------[pt has urinary RETENTION]

A 59M, presents to the ED d/t lower abd pain. The pt has had 2-days of fever, chills, dysuria, & pelvic pain; this morning, he was unable to pass any urine. He has never had similar symptoms before & he has no Hx of chronic urinary or bowel issues. PMHx is sig for hyperlipidemia & CAD. Temp is 102.2F, BP is 140/90, & pulse 98/min. Suprapubic fullness & tenderness are present. DRE shows an edematous & exquisitely tender prostate. The external genitalia are normal. A suprapubic catheter is placed, which drains 800 mL or urine. Labs show: -Leukos----------------------17,8K -Creatinine-------------------1.7 -Leukocyte esterase---------Positive -Nitrites-----------------------Positive WBC's-------------------------10-20 Which of the following is the most appropriate pharmacotherapy for this pt? (Nitrofurantoin OR levofloxacin)

Rupture of an apical alveolar bleb -Pt most likely has a secondary spontaneous pneumothorax → characteristics include: 1) RF --COPD 2) Acute onset SOB, dyspnea 3) Hypoxia 4) Unilateral decreased breath sounds, fremitus & scattered wheezes 5) Unilateral Chest pain 6) Hyper resonance on percussion Diagnostic 1) CXR → visceral pleural line beyond which no pulmonary markings are apparent --Tracheal deviation can occur but it is rare Fluid transudate into the pleural space -Commonly occurs in pts with liver, heart, or renal failure -Symptoms include: dullness to percussion & decreased breath sounds over the effusion (usually in the lower lung fields) -However, the symptoms associated w/ transudate are slow to develop Inflammatory infiltration of the lung -Pts w/ COPD are at increased risk for pneumonia d/t chronic inflammation → however this pt would present w/ pneumonia type symptoms

A 59M, presents to the ED d/t sudden-onset, severe dyspnea & right-sided Chest pain while driving to work. The pt has a Hx of COPD, HTN, GERD, & CKD. He has smoked a pack of ciggs daily for 35-yrs. BP 130/80 & pulse 102/min & regular. Pulse Ox shows 89% on room air. On PE, the trachea is midline. Tactile fremitus & breath sounds are decreased on the right, & scattered wheezes are present on the left. No heart murmurs are heard. He has trace ankle edema. Which of the following is the most likely underlying mechanism of this pt's current condition? (Fluid transudation into the pleural space OR Rupture of an apical alveolar bleb OR Inflammatory infiltration of the lung)

Lower resp tract sampling & culture -Pt presents w/ ventilator-associated pneumonia ---Commonly develops ≥48 hrs after intubation → symptoms include: 1) Fever 2) Purulent secretions 3) Difficulty w/ventilation 4) Leukocytosis Diagnostic steps 1) CXR 2) Lower respiratory tract sampling TMT 1) Empiric antibiotics CT scan of the chest -Used in pts Diagnosed w/ ventilator associated pneumonia who do not respond to initial empiric antibiotic TMT

A 60F, undergoes elective coronary artery bypass surgery & aortic valve replacement. The post-op course is complicated by acute renal failure, A-fib, & pulm edema. On the 3rd post-op day, the pt cannot tolerate attempted extubation & requires re-intubation. On the 5th post-op day, she develops a fever (102F). BP is 110/65, pulse is 110-120/min & irregular, & RR 32/min. CXR shows dense infiltrates in the right side & lower lobes. WBC Ct is 16,2K. w/ 8% bands. Which of the following is the most appropriate next step in MGMT? (CT scan of chest OR Lower resp tract sampling & culture)

FEV1 -The best predictors of post-op lung resection outcomes are: ---1) FEV1 ---2) CO diffusion capacity End-tidal CO2 level -Is indicative of aerobic metabolism -Can be used to monitor correct endotracheal tube placement/to measure the effectiveness of cardiopulmonary resuscitation

A 60M, is evaluated for a right lung mass, which was discovered on a CXR performed one month ago d/t persistent cough. He has had no chest pain, dyspnea, or hemoptysis. The pt has no other sig PMHx & takes no meds. He is an ex-smoker w/a 20-py smoking Hx. PE shows clear lungs, normal heart sounds, & no lymphadenopathy or abd organomegaly. CT shows a 4-cm mass in the right oblique fissure w/no enlarged LNs. Bronchoscopy-guided biopsy reveals SCC. A subsequent PET scan shows no metastatic lesions, & a right pneumonectomy is planned. Which of the following is likely to provide the best assessment of post-op pulmonary morbidity in this pt? (FEV1 OR End-tidal CO2 level)

Antibodies against a plts component -Pt most likely presents w/ HIT from heparin -Presentation includes necrotic like lesions on the abd wall Embolization of cholesterol -Can cause skin findings similar to HIT -However, they skin findings would be seen in the LE's [not the abdomen]

A 62F, is evaluated for worsening skin lesions. She was admitted 6-days ago for elective CABG d/t extensive CAD. The surgery was uncomplicated, & the pt was extubated on hospital day 2. She has recovered well, but yesterday she noticed red patches on her abd that progressed to purple lesions today. The pt has a Hx of non-ST elevation MI, DM II, HTN, & hyperlipidemia. She is receiving low-dose sub-Q heparin for prophylaxis of DVT & has not received any oral anticoagulation. Temp is 98.7F, BP is 130/82, HR 90/min. Cardiopulmonary exam reveals patchy rales & normal heart sounds. The sternal surgical site is healing well. Several large purple/black patches are seen in the periumbilical area, surrounded by erythema. Peripheral pulses are normal. Neuro exam is unremarkable. Which of the following is the most likely cause of the pts skin lesions? (Antibodies against a plts component OR Embolization of cholesterol)

RCC -A) RF ---1) Smoking for 30+ years ---2) Age -B) Signs/symptoms --1) Wt loss --2) Firm, nontender, abd flank mass that moves w/ respiration --3) Hematuria --4) Intermittent fever -C) Labs ---1) Anemia Appendiceal abscess -Can Also presents w/anorexia, & leukocytosis - However, key differences are: ---1) Pt would present w/ pain in RLQ --------[pt has no pain] ---2) Presents w/ HIGH, CONSTANT FEVER -------[pt has INTERMITTENT FEVER] ---3) DOES NOT present as hard mass ---4) NOT ASSOCIATED w/ anemia

A 62F, presents d/t 6 weeks of malaise & intermittent fever. She has also decreased appetite & unintentional wt loss but no bowel/bladder symptoms. PMHx is sig for DM II & HTN, & 10 years ago she underwent total hysterectomy & BSO for early stage endometrial cancer. She has smoked a pack of ciggs daily for 30 years & does not drink. She works as an accountant. Temp is 100.2F, BP is 140/90, pulse is 90/min, & RR are 16/min. On exam, she appears thin & pale. A hard mass is palpated over the right flank area. There is no abd tenderness, & bowel sounds are normal. The liver & spleen are not enlarged. No lymphadenopathy is present. Labs show: -Hb--------------------------10.5 -Leukos----------------------9,8K -Cr, serum-------------------1.2 -Ca---------------------------9.8 Which of the following is the most likely Dx? (Appendiceal abscess OR RCC)

Spinal cord injury -Pt has Hx cervical spine spondylosis (cervical XR findings) → and the she was involved in a rear-end collision & developed subsequent UE weakness, reflex loss, & decreased sensation → this raises suspicion for central cord syndrome → characteristics include: --1) RF's → older pt w/ stenotic cervical spinal canal w/ whiplash like injury --2) Symptoms (mostly UE manifestations) -----UE weakness --------clumsiness -----Pain, temp, & sensory loss ---------Tingling & burning ---------Numbness -----Reflex loss Post-concussion symptoms -Pt would have to present w/ a concussion → but she had no cognitive impairment during or after the accident

A 62F, presents d/t UE weakness & numbness. Three days ago, the pt was involved in a MVC in which a truck rear-ended her vehicle. The pt did not hit her head of loose consciousness but experienced a tingling & burning sensation in all extremities, which she attributed to the stress of the accident. Since then, she has had numbness in her UE's & clumsiness of her hands. She reports difficulty picking up objects but has had no leg weakness or bowel/bladder disturbance. The pt is upset that the other driver refused to accept fault but reports no other ongoing stressors. She has a PMHx sig for HTN & OA. Vitals are normal. Neck & back exam show no deformity or tenderness. There is weakness of hand grip bilaterally, & the triceps reflex is decreased on both sides. Decreased pinprick sensation is present on the fingers. The remainder of the neuro exam is normal. Cervical spine XR reveals no vertebral dislocation or fracture, but diffuse spondylitis changes are present. Which of the following is the most likely cause of this pts symptoms? (Post-concussion syndrome OR Spinal cord injury)

Surgery -The MRI shows a cerebellar hemorrhage → Complication of about 10% of cases of ICP A) MGMT → based on whether or not the hemorrhage has extended into other areas --1) No extension signs -----a) Headache, nausea, vomiting, meningismus --B) MGMT ------1) Stabilization (airway, breathing, CV system) ------2) Reversal of anticoagulation ------3) BP MGMT ------4) Elevation of the bed to decrease ICP --2) Signs of extension -----a) Vertigo -----b) Dysarthria -----c) Nystagmus -----d) Ataxia --------Ipsilateral truncal (cerebellar vermis) --------Ipsilateral limb (Cerebellar hemispheres --C) MGMT -------1) Urgent surgical decompression → especially in pts w/ signs of: -----------a) Neuro deterioration (progressive lethargy & coma) -----------b) Radiologic evidence of a bleed >3-cm, brainstem compression, or obstructive hydrocephalus Loop diuretics -No role in MGMT of cerebellar hemorrhage

A 62F, presents to the ED by her family d/t dizziness. She was unable to get out of bed & go to the bathroom this morning & has had headache & nausea. The pt has a Hx of migraine headaches & HTN. 6-mnths ago, she had an episode of transient right-sided vision loss & was prescribed aspirin. Other meds include amlodipine, & hydrochlorothiazide. Temp 98,1, BP 156/72, & pulse is 88/min. BMI is 33. On PE, the pt moves all extremities, & deep tendon reflexes are symmetric. CT scan of the head is shown. The pt is admitted to the hospital. An hour after admission, she appears lethargic but follows simple commands. She had a single episode of vomiting in the ED. Which of the following is the best next step in MGMT? (Loop diuretics OR Surgery)

Metastatic disease to liver -Pt most likely has undiagnosed colon cancer → Characteristics include: --1) Metastasis is Most common cause of a liver mass ----(Much more common than primary liver cancer) ----Multiple nodules are usually seen → However, solitary nodules are not uncommon Hepatic adenoma -Usually seen in young women -Usually asymptomatic; may present w/ RUQ pain

A 63M, presents d/t anorexia & wt loss for the past 2 months. He has no chest pain, abd pain, cough, or fever. PMHx is sig for HTN & latent TB. He doses not smoke, drink, or use drugs. Temp is 99.5F, BP 124/80, Pulse 78/min, RR 14/min. PE reveals mucosal pallor & mild hepatomegaly. The abd is soft, non-distended, & non-tender. Cardiopulmonary exam is normal. Fecal occult blood test is positive. Labs show: -Hb------------------------9.8 -MCV----------------------72 -Plts-----------------------276K -Leukos--------------------6,7K -Total bili-------------------1.3 -ALP------------------------190 -AST------------------------32 -ALT-------------------------38 US reveals a solitary liver lesion measuring 2x3-cm. What is the most likely Dx? (Metastatic Disease to liver OR Hepatic adenoma)

Pressure necrosis - Pt most likely has a pressure (decubitus) ulcer -A) RF ---1) Nursing home pt ---2) Hemiparesis -B) Characteristics ---1) Most prominent in sacrum, ischial tuberosities, malleoli, heels, & 1st/5th metatarsal head Venous insufficiency -A) RF ---1) Occurs in the setting of LE edema & stasis dermatitis ------[Pt has no Hx of either] -B) Characteristics ---1) Most commonly seen at pre-tibial/above medial malleolus Arterial insufficiency -A) RF ---1) Occurs in setting of PAD ------[pt has no Hx of PAD] -B) Characteristics ---1) Most often occurs in most distal parts of body (tips of toes)

A 64F, is evaluated in a nursing home for a foot ulcer. 3-mnths ago, she had a cerebrovascular accident & now has residual hemiparesis w/ contractures. PMHx is sig for HTN, DM, hypercholesterolemia, CAD, & mild dementia. Temp 98.9F, BP 142/63, pulse 62/min. She is alert & oriented to person but not to time or place. There are no new focal neurological deficits. Skin exam shows ulcerations on the left foot w/mild erythema (shown). Which of the following is the primary cause of this pts skin ulcer? (Pressure necrosis OR Venous insufficiency OR Arterial insufficiency)

Laryngopharyngoscopy -A) Signs/symptoms (Of head & neck Malignancy) include: ---1) Persistent symptoms (<2 weeks) ---2) Palpable cervical LN (>1.5cm) → OFTEN FIRST SIGN ---3) Firm neck mass -B) Associated RF ---1) Smoking/alcohol use ---2) No Hx of preceding infection -----(Palpable LN would be EXPECTED in pts w/ previous infection) -D) Diagnostic ---1) Laryngopharyngoscopy -------(The vast majority of cervical node SCC arises from mucosal surfaces of the head & neck (nasopharynx, oral cavity, oropharynx, larynx) Colonoscopy -These malignancies can spread to the left supraclavicular LN, via the thoracic duct -----[pt has a palpable CERVICAL LN] - However most are adenocarcinomas -----[pt has squamous cell carcinoma]

A 64F, presents for an eval of a painless neck lump. The pt first noticed right-sided neck swelling a month ago. She has not had fevers, chills, cough, SOB but has decreased appetite & wt loss. PMHx is sig for obesity, DM II, & non-alcoholic fatty liver disease. The pt has smoked a pack of ciggs daily for the past 29 years. She drinks alcohol socially. Vitals are w/in normal signs. PE shows a 2-cm, hard, non-tender cervical LN close to the right submandibular gland. There is no other lymphadenopathy. The lungs are clear to auscultation & heart sounds are normal. The abd is soft & non-tender; mild hepatomegaly is present. The remainder of the exam is normal. FNA of the LN reveals Squamous cell carcinoma. Which of the following would be most helpful to establish the primary source of this pt's malignancy? (Colonoscopy OR Laryngopharyngoscopy)

Water-soluble contrast esophagography -Pt most likely has esophageal perforation → indicated by: --1) Setting of Recent esophageal instrumentation (TEE) --2) Severe chest pain (inter-scapular) --3) Fever, Tachycardia --4) Widened mediastinum on CXR -B) MGMT ---a) Diagnostic tests -------1) Water-soluble esophagography or water-soluble CT ----------a) Barium Contrast is more sensitive, but can elicit a granulomatous inflammatory response → used when water-soluble is inconclusive ---b) TMT -------1) Emergent surgery -------2) IV antibiotics & PPI's CT angiogram of the chest -Would be used if AAA was suspected → however AAA is associated w/ ---1) New aortic regurg murmur ---2) Asymmetric BP ---3) Does NOT typically present w/ a fever ------[Pts fever & recent instrumentation make perforation much more likely]

A 65F, presents d/t feeling her heart race, even while she is at rest. She has had multiple episodes like this over the past 2-wks, lasting anywhere from a few mins to a couple of hours & associated w/ lightheadedness. The pt's other medical Hx include obesity, DM II, HTN, & GERD. She has smoked a pack of ciggs daily from age 16-35. Vitals are normal except for an irregularly irregular pulse. The pt is Dx'd w/ A-fib & given anticoagulation & rate control meds. In preparation for cardioversion, she undergoes transesophageal echocardiography, which rules out a mural thrombus. That evening the pt develops severe chest pain & inter-scapular back pain. Temp is 102F, BP is 100/70, pulse is 108/min. The pt is in distress & restless. Heart & lung sounds are normal. ECG shows sinus tachycardia w/out ST segment changes. CXR reveals a widened mediastinum. Stool testing for occult blood test is neg. Which of the following is the best next step in Dx? (CT angiogram of the chest OR Water-soluble contrast esophagography)

CT scan of abdomen - Pt has signs & RF's concerning for pancreatic cancer including: --1) RF's ----a) Recent DM II Diagnosis -------25% of pts w/ pancreatic cancer are Dx'd w/ DM <2 years prior to discovery of tumor -------Atypical DM (thin, older pt) ----b) Age ----c) Smoking Hx --2) Wt loss --3) Constant (gnawing) epigastric pain Upper GI series -would be used to investigate a pt w/ dysphagia

A 65M, presents d/t 4-wks of persistent epigastric pain & nausea. The pain is constant & does not improve or worsen w/ activities, position, or diet. The pt lost approximately 9.9-lbs over this period. He has no fever, vomiting, constipation, or diarrhea. He was Dx'd w/ DM II 6-months ago, which is controlled w/ a low-carb diet. The pt has never had surgery & has no other chronic med conditions. He has a 45-py smoking Hx. He does not drink or use drugs. Temp is 97.7F, BP is 122/72, pulse is 68/min, RR 16/min. Mucous membranes are moist. No lymphadenopathy is present in the cervical/axillary chains. Bowel sounds are present, & the abd is soft w/ mild epigastric tenderness to deep palpation. No hepatosplenomegaly or shifting dullness is present. The remainder of the exam is normal. CBC, serum chemistry, & liver functions studies are all normal except for a fasting blood sugar of 130. Which of the following is the most appropriate next step in MGMT? (CT scan of abdomen OR Upper GI series)

Polymicrobial infection by contiguous spread -Foot infections are common in pts w/ DM → they can be either deep/superficial Deep infections -Most likely polymicrobial -Characteristics include: ---Long standing (>1-2 wks) ---Systemic symptoms ---Larger ulcer size ---↑ ESR ---Osteomyelitis (Presence/palpation of bone in ulcer) Superficial infections -1) Key differences are: ---a) NO osteomyelitis ---b) smaller size ---c) Most likely monomicrobial

A 65M, presents d/t a week of low-grade fever & dull pain in his right foot. He developed a small ulcer on his right foot 2-mnths ago that slowly worsened & never healed. He does not recall any precipitating trauma. The pt has a PMHx sig for DM, HTN, hyperlipidemia, CAD, & PAD. Her smoked a pack of ciggs daily for 35 years but quit 5-yrs ago. Temp 101F, BP 130/80, pulse 88/min. PE shows a 2x2 ulcer on the plantar surface of his right forefoot surrounded by a halo of erythema & edema. There is no visible bone at the base of the ulcer, & metal probing does not contact bone. Pedal pulses are decreased bilaterally. Right foot XR reveals cortical erosion of the first metatarsal. Which of the following is the most likely mechanism for bone infection? (Monomicrobial infection by contiguous spread OR Polymicrobial infection by contiguous spread)

Renal US w/ doppler -Pt presents w/ recurrent flash pulmonary edema --(pulmonary crackles, distended neck veins, sudden-onset SOB, LE edema) → despite a normal EF → in association w/ severe HTN this suggests RAS -Secondary HTN (hyper-aldosterone) associated labs include: --1) Hypokalemia --2) Elevated bicarb MGMT 1) Must check kidneys w/ US with doppler Kidney biopsy -Only if RCC was suspected → pt has no Cancer-Related symptoms

A 65M, presents d/t sudden-onset SOB. He could not lie flat d/t difficulty breathing. There is no associated chest pain. Six months ago, the pt was hospitalized w/ similar symptoms & responded well to diuretic therapy. PMHx is sig for HTN, CAD, & TIA's. The pt has undergone several percutaneous coronary interventions for stable angina in the past & had a right carotid endarterectomy 2 years ago. He has good adherence to his daily meds but smokes a pack of ciggs daily. BP is 210/100, pulse is 104/min & regular, & RR are 22/min. O2 sat is 86% on room air. Exam shows distended neck veins & bilateral lung crackles. The abd is soft, obese, & nontender. The extremities have no edema. Labs show: -Hb-------------------------13 -Plts------------------------240K -Leukos--------------------8,2K -K+--------------------------3.1 -Bicarb----------------------30 -BUN------------------------30 -Cr, serum------------------2.0 -Ca+------------------------8.4 -Glucose-------------------100 -total protein---------------6 -Albumin--------------------4.3 -AST (SGOT)----------------12 -ALT (SGLT)-----------------24 Urine -Protein---------------------trace -Blood-----------------------neg -Glucose---------------------neg -WBC's-----------------------1-2 -RBC's------------------------1-2 -Casts------------------------n/a -Crystals----------------------n/a Cardiac troponin levels are not elevated. ECG shows sinus tachycardia w/ LV hypertrophy. Echo reveals LV hypertrophy, preserved LV EF, & no regional wall abnormalities. The pt improves markedly w/ diuresis & anti-HTN meds. Which of the following is the next best step in MGMT? (Renal US w/ doppler OR Kidney biopsy)

Pneumothorax -A) RFs include: ---1) Iatrogenic ------a) PEEP/mechanical ventilation ----------(Pt presents w/ hypoxemia w/ elevated peak & plateau pressures, after PEEP) ------b) Thoracocentesis ------c) Errors in subclavian line placement ---2) Underlying pathology ------a) COPD ------b) Trauma ------c) Infection -B) Signs/Symptoms → P-THORAX ---P → pleuritic chest pain ---T → Tracheal deviation ---H → Hyper-resonance ---O → Onset abrupt/acute ---R → Reduced breath sounds ---A → Absent fremitus ---X → CXR findings → collapse ---a) Other signs ------a) Coughing, wheezing, worsening SOB Right mainstem bronchus intubation -Can also present w/ hypoxemia - However, key differences are: ---1) Decreased breath sounds in the left, unventilated lung -------[not right lung, as in this pt] Pulmonary embolism -A large PE can cause acute hypoxemia - However, key differences are: ---1) Usually presents in pts w/ previous Hx of DVT or immobilization ------[pt does not have this in Hx] ---2) WOULD NOT cause elevated ventilatory pressure or ---3) DOES NOT present w/ decreased unilateral breath sounds

A 65M, w/ COPD is hospitalized d/t worsening SOB, coughing, & wheezing. On arrival to the ED, the pt was in severe respiratory distress w/bilateral wheezing. CXR revealed hyper inflated lungs w/out infiltrates. Endotracheal intubation was performed & mechanical ventilation initiated. Inhaled bronchodilators, systemic glucocorticoids, & empiric antibiotics were administered. Respiratory status gradually improved, & the pt was admitted to the ICU. Now, 45-mins later, progressive hypoxemia w/elevated peak & plateau pressure has developed. BP is 150/86, pulse is 108/min, RR 22/min. O2 sat is 88% despite an increase in FiO2 to 70%. There is no wheezing, but breath sounds are decreased on the right. Tracheal suctioning yields no mucus. Which of the following is the most likely cause of the deterioration in this pt's condition? (Pneumothorax OR Pulmonary embolism OR Right mainstem bronchus intubation)

Electrolyte imbalance -Pt most likely presents w/Ogilvie syndrome (acute colonic pseudo obstruction) -A) Signs/Symptoms --1) Severe, abrupt abd distention ------a) Pain, vomiting, inability to pass stool ------b) Tympanic abd w/ hypoactive bowel sounds -B) MGMT ----a) Diagnostic test -------1) Abd CT ----------a) Colonic dilation w/ no anatomic obstruction -C) Etiologies include ----1) Electrolyte derangement (D/t recent diarrhea) ----2) Major surgery/traumatic injury/severe infection ----3) Meds (opiates & anti-cholinergic) ----4) Neurologic disorders (dementia/stroke) Colonic endoluminal mass --[Symptoms of obstipation & abd distention tend to develop less abruptly] --[CT would show a mass] Colonic hypo-perfusion -Leads to ischemic colitis → which presents w/ hematochezia & abd pain --[Colonic dilation not typical]

A 67M, presents to the ED d/t abd discomfort & distention. 5 days ago, he began having large-volume watery stools 6-8 times a day after eating at a new restaurant. He also had had associated mild abd cramps & nausea but no fever, chills, vomiting, or bloody stools. His wife, who had the same food, also experienced similar symptoms. He self-treated w/ increased oral fluids & the stool frequency & consistency began to improve. However, his BM's abruptly stopped yesterday, & the diffuse abd discomfort & distention worsened. PMHx is sig for HTN, CAD, & HF treated w/diuretics. Temp is 98.6F, BP is 116/72, pulse is 102/min. PE shows dry mucous membranes. The abd is distended, tympanic, & mildly tender to palpation w/no guarding or rebound tenderness. Bowel sounds are decreased. DRE reveals an empty rectal vault w/no evidence of blood or melena. Abd CT shows diffuse colonic dilation w/ non-dilated small bowels. Leukocyte Ct is normal & testing for C.diff is neg. Which of the following most likely contributed to this pts current condition? (Colonic endoluminal mass OR Colonic hypo-perfusion OR Electrolyte imbalance)

Instability of the atlantoaxial joint -Pt most likely has subluxation → in the setting of her RA she develops weakness, paresthesias, & hyperreflexia after intubation → symptoms include: 1) Cervical pain that radiates to the occiput 2) Slowly progressive, spastic paraparesis characterized by weakness involving the UE & LE 3) Hyperreflexia 4) Sensory changes Diagnostic - MRI (C1 & C2) Critical illness-related axonal injury -Describes critical illness polyneuropathy → usually a complication of sepsis -Common cause of weakness after a prolonged ICU stay -Peripheral nerves are usually affected causing LMN symptoms

A 68F, presents for eval of LE weakness. The pt was hospitalized d/t pneumonia & septic shock a month ago. She was intubated for respiratory failure at the time of admission & was treated in the ICU for several days. After adequate improvement in her respiratory status, she was discharged to a rehab facility. Since that time, the pt has had worsening LE weakness that has failed to improve despite PT. She has also had frequent occipital headaches & numbness & tingling in the hands & feet. The pt has a Hx of RA, HTN, hypothyroidism, & right total knee replacement.. She is a former smoker w/a 20-py smoking Hx & does not drink. BP is 136/82 & pulse 72/min. PE shows chronic joint deformities in both hands, clear lungs, normal heart sounds, & a nontender abd. Neuro exam reveals decreased LE muscle strength, hyperactive knee reflexes, & decreased proprioception in the feet. Rapidly flicking the nail of the middle finger elicits flexion of the ipsilateral thumb. Which of the following is the most likely cause of this pts current symptoms? (Critical Illness-related axonal injury OR Instability of the atlantoaxial joint)

Echo -Pt presents w/ post-op cardiac tamponade → presentations includes: --1) RF's -----Complication of CABG --2) Cardiogenic shock symptoms -----Hypotension (Tachycardia) -----Reduced urine -----Cool extremities --3) Classic triad ----A) Hypotension ----B) Elevated JVP ----C) Distant heart sounds Tests 1) Pulmonary Catheter → Elevation & equalization of intra-cardiac diastolic pressures Diagnostic 1) Urgent Echo Dobutamine infusion -Used in LV failure (for inotropic acute HF)

A 68M, is recovering in the ICU after a CABG. Two weeks ago, he was seen in the office for progressive exertional angina & SOB. The pt underwent coronary angio, which indicated severe extensive coronary artery disease. After discussion about the risks & benefits, he agreed to proceed w/ open heart surgery. The pt has a FMHx of CAD but no sudden cardiac death, HF, or bleeding. On post-op day 1, the nurses report that the pt is confused & has had reduced urine output. Temp is 96F, BP 80/50, pulse 118/min & regular, RR are 22/min. There is no drainage from his sternal wound, & there has been no drainage in the chest tube over the last 4-hrs. The lung fields are clear. No heart murmurs are appreciated. The abd is soft. His extremities are cool. ECG shows sinus tachycardia & non-specific T-wave changes. Pulmonary artery catheter readings are: -RA------------20 (N 2-8) -RV------------35/20 (N 15-30/2-8) -PCWP--------20 (N 6-12) Which of the following is the most appropriate next step in MGMT? (Dobutamine infusion OR Echo)

Surgical exploration & sternal fixation -Pt presents w/ sternal dehiscence → a complication of cardiac surgery → presentation includes: --1) Chest wall instability → "Clicking" w/ chest mvmt TMT 1) Sternal fixation (surgical emergency)

A 68M, presents for a follow-up after recent CABG surgery. He describes anterior chest "clicking" & "shifting" when he bends or moves his upper extremities. The pt has no chest pain, dyspnea, fever, chills, or abd pain. He underwent uncomplicated surgery w/internal thoracic artery harvesting 6-days ago for severe CAD. The pt is afebrile, & vital signs are w/in normal limits. PMHx is sig for DM, obesity, & COPD. Exam reveals a midsternal wound that is clean w/well-approximated sutures & minimal serous discharge at the lower edge but w/out erythema. On palpation the sternum appears to be rocking & clicking w/pt coughing. Which of the following is the best MGMT? (Clinical observation only OR Surgical exploration & sternal fixation)

Arterial embolus -Embolus presents w/ 6P's 1P) Pain 2P) Pallor 3P) paresthesias 4P) Pulselessness 5P) Poilkilothermia (cool extremity) 6P) Paralysis -Most present w/ sudden development of symptoms Etiology 1) Embolus from cardiac/arterial source 2) Iatrogenic 3) Direct blunt trauma Arterial thrombosis -Usually develops at the site of an atherosclerotic plaque in pts w/ preexisting PVD → symptoms are less severe compared w/ acute embolus -Also, pulses would be diminished in both extremities

A 68M, presents to the ED d/t right let pain for the past several hours. The pt was resting at home when he suddenly had severe pain in the right leg; the leg has since become numb. He has never had similar symptoms & has no Hx of trauma, fever, or chills. The pt recently suffered an acute anterior wall MI resulting in cardiogenic shock & is currently undergoing cardiac rehab. PMHx is sig for DM II & hyperlipidemia. Temp 98.1F, BP 120/70, pulse 90/min & regular, RR 16/min. Cardiopulmonary exam is unremarkable. Compared to the left leg, his right leg appears pale & cool to the touch. In the right LE, the popliteal pulse is normal, but more distal pulses are not palpable. Pulses in the other extremities are normal. Neuro exam shows loss of sensation over the dorsum of the foot & mild weakness w/ dorsiflexion. Which of the following is the most likely cause of this pts symptoms? (Arterial embolism OR Arterial thrombosis)

Clinical observation -Pleural effusions occur in almost 50% of all CABG pts → small ones can be managed conservatively → criteria include: --1) Small-to-moderate size & not enlarging --2) Early onset (post-op day 1/2) --3) Not associated w/ resp symptoms Oral antibiotics -Appropriate in pts w/pleural effusions d/t pneumonia or HF (pts w/ fever)

A 72M, is evaluated on day 2 after a CABG for severe CAD. He has no new symptoms except for mild sternal wound discomfort excerpted by deep breathing. The pt's surgery was uncomplicated; he was extubated on post-op day 1. PMHx is sig for DM II & hyperlipidemia. He is afebrile. BP 122/75, pulse 75/min & regular, RR 16/min. O2 sat is 98% on room air. The sternal wound appears to be healing normally. Breath sounds are decreased on the left base. There is no abd distension or peripheral edema. CXR reveals a small, left-sided pleural effusion. Leuko's Ct is 9,2K & serum Cr is 1.1. Which of the following is the best next step in MGMT? (Clinical observation OR Oral antibiotics)

Oropharyngeal aspiration -Characteristic that support aspiration includes: -1) Hx of parkinsons (swallowing dysfunction) Abscess symptoms include: 1) Long-standing cough productive of yellow sputum 2) Systemic symptoms 3) Leukocytosis 4) Cavitation on CXR (diagnostic) 5)Foul-smelling sputum 6) Coughing during eating w/ worsening dysphagia Histoplasma -also presents w/ a chronic presentation including fever, & wt loss, is endemic to the Ohio river valley & would present w/ CXR findings -However, Key differences include: ----1) CXR would show bilateral reticular infiltrates & calcifications -----[Pt has a CAVITATION, which rarely occurs w/Histo] ---2) OTHER KEY SYMPTOMS MISSING, including ------a) Hepatomegaly ------b) Lymphadenopathy ------c) Pancytopenia ------d) ± oral/GI lesions ------e) RASH → erythema nodosum/erythema multiforme

A 72M, presents d/t 4-wks of worsening productive cough. The pt has also had malaise & night sweats & has lost 6.6-lbs during this period. His wife states that he had occasional nonproductive cough in the past, particularly while eating, but it is not persistent & productive of abundant yellow sputum. The pt has a Hx of HTN & Parkinson's. He has a remote 5-py smoking Hx & doesn't drink. He is a retired schoolteacher, lives in Ohio, & has not traveled or been exposed to sick contacts recently. Temp is 100F, BP is 126/68, pulse is 88, RR 14/min. Pulse Ox is 96% on room air. On exam, the pt appears frail w/resting tremor of both hands. Lung auscultation reveals scattered rhonchi but no wheezing. Heart sounds are normal. Labs show: -Hb-------------------------11.4 -MCV-----------------------92 -Plts-------------------------420 -Leukos----------------------13,1K CXR reveals a density w/cavitation in the right lower lobe. Which of the following is the most like cause of this pt's condition? (Histoplasma OR Oropharyngeal aspiration)

Cystoscopy -Pt is an older male w/unexplained hematuria & a smoking Hx, so he must be investigated for RCC/bladder cancer Urine culture -Cystitis usually presents w/ acute onset of hematuria, lower urinary tract symptoms, & abnormal urinalysis (significant pyuria & RBC's) -However, pts duration (2-mnths) & absence of significant pyuria make this less likely

A 74M, complains or urinary frequency over the last 2-mnths. He wakes up to use the bathroom several times during the night. He also complains of mild straining during urination. Over the last few wks, he has had 2 episodes of bloody urine that cleared spontaneously. The pt otherwise considers himself in good health. He had a right knee replacement for severe OA 7 yrs ago. His father died of colon cancer. He has a 30-py smoking Hx but quit 15-yrs ago. Rectal exam shows an enlarged & smooth prostate w/no nodules, & normal rectal sphincter tone. Serum creatinine is 0.8, PSA is 3.8 (N <4). Urinalysis shows 0-1 WBC's, 3 RBC's, & trace protein. Which of the following is the next best step in MGMT? (Cystoscopy OR Urine culture)

Urgent cholecystectomy -Pt has emphysematous cholecystitis → characterized by: --1) Fever --2) RUQ pain --3) Gas (crepitus) in the gallbladder/surrounding tissue RF's 1) DM 2) Vascular compromise 3) Immunosuppression TMT 1) Urgent cholecystectomy → The presence of gas in the gallbladder makes this an emergency (usually caused by Clostridium and can progress to gangrenous cholecystitis & gallbladder perforation) Elective cholecystectomy 2-wks after recovery -would be correct if pt was high risk AND had a regular cholecystitis

A 75F, presents to the ED d/t fever, chills, abd pain, nausea, & vomiting for the past 2-days. She did not eat anything unusual, & no one else at her home is sick. The pt has no diarrhea, dysuria, or urinary frequency. PMHx is sig for DM II, HTN, Obstructive sleep apnea, & CAD. Temp is 102F, BP is 130/80, pulse is 104/min, RR 18/min. O2 sat is 94% on room air. BMI is 32. On PE, the pt appears acutely unwell. The oropharynx has dry mucous membranes, & dentition is poor. Cardiopulmonary exam shows normal lung & heart sounds. Abd is mildly distended. There is marked tenderness over the epigastrium & RUQ w/guarding. Bowel sounds are decreased. The extremities have no cyanosis or edema. Labs show: -Hb-----------------------10.9 -Leukocytes-------------18,7K (10% bands) -Total Bili-----------------2.0 -Indirect bili--------------1.4 -Lipase--------------------Normal -BUN---------------------34 -Creatinine---------------1.4 -Blood sugar-------------340 Abd US reveals thickening of the gallbladder wall, pericholecystic fluid, & multiple hyper echoic calculi. Air is noted in w/in the gallbladder wall, & the CBD is not dilated. IV fluids, insulin, analgesics, & broad-spectrum antibiotics are initiated. Which of the following additional intervention is most appropriate in MGMT? (Elective cholecystectomy 2-wks after complete recovery OR Urgent cholecystectomy)

Keratoacanthoma -Presents as: --1) Rapidly growing nodule w/ulceration & a central keratin plug --2) Frequently regress & may resolve spontaneously over several months --3) Most common in fair-skinned people & often in areas of UV exposure --4) Complication → SCC, so must do Biopsy Cutaneous horn -Keratinaceous projection that are associated w/ benign (seborrheic keratosis) or malignant (SCC) -Typically elongated/conical -Do not undergo rapid regression

A 77F, presents d/t a skin lesion. The pt first noticed it a few wks ago when it grew rapidly & became painful. However, since she scheduled her appt 3-wks ago, the lesion has become somewhat smaller & less painful. Skin exam is shown. What is the most likely Dx? (Cutaneous horn OR Keratoacanthoma)

Pancreatic duct injury -Can result from BAT → - PE findings may be non-specific & initial serum amylase & lipase levels may be normal → so injury should be suspected in pts w/ NORMAL ABD CT's & 1) Persistent abd discomfort 2) Persistent nausea/emesis 3) Increasing amylase over serial msmts 4) Peri-pancreatic fluid collection Diagnostic 1) Cholangiopancreatography Hepatic laceration -Would be seen on abd CT -----[pt has NORMAL CT]

A previously healthy 19M, presents to the ED d/t persistent abd discomfort. The pt previously came to the ED 3 days ago after he fell while mountain biking. He did not lose consciousness & recalls hitting the handlebars prior to flipping & landing on his back. His trauma workup- which included CT of the chest, abd, & pelvis- was neg, & he was discharged. Since then, the pt has had persistent upper abd discomfort & nausea, along with an episode of non-bilious emesis. Temp is 100.6F, BP is 104/62, & pulse is 108/min. the lungs are clear to auscultation, & the heart sounds are normal. Ecchymosis is present across the upper abdomen, which is tender to palpation w/ voluntary guarding. Bowel sounds are decreased. Bedside US reveals a large amount of free fluid in the upper abd. Which of the following is the most likely cause of this pts condition? (Hepatic laceration OR Pancreatic duct injury)

Mucinous infiltration -Carpal tunnel associated w/hypothyroidism is caused by soft tissue enlargement (mucopolysaccharides) Edema (fluid accumulation) -Cause of carpal tunnel in pregnancy Tendon enlargement & synovial edema -Cause of carpal tunnel in acromegaly

A pt presents w/carpal tunnel. She has a Hx of hypothyroidism. Which of the following is most likely responsible for this pt's hand symptoms? (Mucinous infiltration OR Edema (fluid accumulation) OR Tendon enlargement & synovial edema)

Toxic shock syndrome -Characteristics include: 1) Recent Hx of nasal packing (does not have to be tampon use) followed by rapid onset of: ---Rash (Diffuse macular erythroderma similar to sunburn) ---Fever ---Hypotension (dizziness) ---Diarrhea, ---Vomiting ---Diffuse myalgias Labs 1) Increased blasts (immature neutros) 2) Thrombocytopenia Meningococcemia -Skin findings include petechial rash that progresses to ecchymosis, bullae, vesicles, & ultimately gangrenous necrosis -Pts also presents w/ fever, nausea, vomiting, severe myalgias, shock - However, key differences are: ---1) Typically presents w/ MENINGEAL SIGNS (i.e., photophobia) --------[pt does not have photophobia] ---2) Nasal packing is NOT A RF for meningococcemia

An 18F, presents to the ED w/severe vomiting, diarrhea, & dizziness that started this morning. 3-days ago, she had nasal bleeding that required anterior packing in the ED. Yesterday, she had pain & a sensation of pressure under her left eye & scant blood-tinged nasal discharge. The pain was relieved by acetaminophen & ibuprofen. The pt, who has no sick contacts, has a Hx of asthma & allergic rhinitis. Her last menstrual period was 3-wks ago. She does not smoke, drink, or use drugs. Temp is 102.2F, BP is 90/60 supine & 66/45 standing, pulse 120/min, RR 23/min. She appears alert but restless. On exam, there are diffuse, confluent erythematous macules on her trunk & extremities. The oropharynx appears hyperemic. Labs show: -Plts---------------------55K -Leukos-----------------9,5K --Bands-----------------30% Which of the following is the most likely Dx? (Meningococcemia OR Toxic shock syndrome)

Tearing of the bridging veins -Pt has an acute subdural hematoma (Crescent shaped hyperdensity in lower left corner of the MRI) → most common cause is the tearing of the bridging veins Hyalinosis of small vessels -Mainly caused by HTN -Lesion would be located in deep Brain structures (internal ganglia, thalamus, internal capsule)

An 80F, presents to the ED d/t progressive confusion, generalized weakness, & unsteadiness for the past several days. The pt had a right femoral fracture 4 months ago & underwent hip surgery. She has been using a walker to ambulate since coming home from a physical rehab center 3-wks ago. She has had no fever, chest pain, vomiting, diarrhea, or dysuria. PMHx is sig for HTN, hyperlipidemia, CAD, TIA, & persistent A-Fib. Meds include aspirin, warfarin, metoprolol, & a statin. Temp is 99F, BP is 140/90, pulse 72/min & irregular, RR 14/min. CT is shown. Which of the following is the most likely cause of this pts condition? (Hyalinosis of small vessels OR Tearing of the bridging veins)

Small bowel herniation -Pt most likely has an SBO → characteristics include: 1) Progressive abd pain w/nausea, vomiting (leading to dehydration [dry mucous membranes & elevated creatinine]) 2) Abd distension 3) High-pitched bowel sounds 4) Air-fluid levels on Abd XR Pathophysio -Usually d/t to small bowel herniation/strangulation Groin hematoma -Can cause extrinsic compression -More common in anti-coagulated pts -Pts typically has a Hx of trauma -Anemia & ecchymosis would be expected

An 80F, presents w/ progressively increasing abd pain, nausea, vomiting, & an inability to keep food down since yesterday. PMHx is sig for chronic paroxysmal A-fib, for which she takes metoprolol & apixaban, & she has never had any surgeries. The pt does not smoke, drink, or use drugs. Temp 98.8F, BP 150/80, pulse 96/min, RR 16/min. Oral exam shows dry mucous membranes. Cardiopulmonary exam reveals no abnormalities. The abd is distended & tympanic, but there is not tenderness, rebound, or rigidity. Bowel sounds are increased & high-pitched. Rectal exam shows no stool in the rectal vault & no masses. Fullness & tenderness are noted in the right groin area. The extremities are w/out cyanosis, clubbing, or edema. Labs show a Hb of 13.2, BUN 38, Creatinine 1.6. Abd XR shows distended bowel loops w/air-fluid levels. Which of the following most likely predisposed this pt to her current condition? (Groin hematoma OR Small bowel herniation)

Air embolism -Pt develops rapid decompensation & cardiac arrest following central vein catheter placement → she most likely experienced a venous air embolism → characteristics include: -1) RF's ---Trauma ---Surgeries (neurosurgical, otolaryngological) ---Central vein catheters ---Pulmonary barotrauma Pathophysio -If the embolism is large → It can lodge into the RV outflow tract/pulmonary arterioles leading to sudden: ---1) Sudden-onset dyspnea & respiratory distress ---2) Obstructive shock ---3) Cardiac arrest (Pulseless electrical activity) ---4) Hypoxemia Cardiac tamponade -Can cause symptoms via obstructive shock - However, key differences are: ---1) it would be unlikely to cause sudden ventilation-perfusion mismatching & hypoxemia w/ clear lungs ---2) Presents w/ distant heart sounds ------[Pt does not have symptom] --3) Not a common cause of PEA

An 85F, is admitted to the hospital d/t suspected acute pyelonephritis. On arrival, BP is 100/80, pulse is 102/min, & RR are 16/min w/ normal O2 sat. An internal jugular venous catheter is being placed for fluid resuscitation & antibiotics. During the procedure, the pt becomes restless & has SOB. Repeat BP is 80/50, pulse is 122/min & regular, RR are 30/min. O2 sat is 85% on room air. Pupils are equal & reactive to light. The trachea is midline, & there is no stridor. Breath sounds are equal & normal. Neuro exam reveals no gross motor or sensory deficit. While being evaluated, the pt develops pulseless electrical activity. Which of the following is the most likely Dx? (Cardiac tamponade OR Air embolism)

A 28M, w/ a Hx of spinal cord injury presents to the ED d/t headache, restlessness, & excessive sweating for the past several hours. A year ago, he was involved in a MVC that caused a C6 spinal cord injury, leading to paraplegia & neurogenic bladder. He has no other sig PMHx. Temp is 96.8F, BP is 170/100, pulse is 52/min. PE shows facial flushing & excessive perspiration from his face & neck. The lungs are clear to auscultation & heart sounds are normal. The abd is soft w/a mass palpable in the lower abd. A suprapubic catheter is present. Neuro exam demonstrates absence of pain & temp sensation below the nipples & paraplegic & spastic LE's unchanged from previous examination. Further eval reveals occlusion of the urinary catheter & urinary retention. The urinary catheter is changed, which leads to rapid improvement in his symptoms & HTN. Which of the following best explains the observed findings in this pt? (Autonomic dysreflexia OR Peripheral autonomic neuropathy)

Autonomic dysreflexia -Autonomic dysreflexia is a complication of spinal cord injuries above T6 (pt had injury at C6) → signs/symptoms incl ude: --1) Severe HTN --2) Flushing --3) Diaphoresis --4) Bradycardia --5) Urinary retention ---6) Nasal congestion Peripheral autonomic neuropathy -Describes diabetic neuropathy


Ensembles d'études connexes

Period 5 1750 CE - 1914 CE Crash Course

View Set

Ch 7 Older Adult health assessment

View Set

mastering geology chapter 17 groundwater

View Set